Talk to our experts

1800-120-456-456

Diffraction of Light - Young’s Single Slit Experiment

  • JEE Advanced
  • Single Slit Diffraction

ffImage

Single Slit Diffraction of Light

Diffraction is the line of study that helped in the development of precise spectrometers, hence helping Astronomy take great leaps. This was possible in the early nineteenth century when two great scientists were struggling in their lives separately in Italy and Germany. These two weak youngsters were still struggling to get educated. Fraunhofer was the son of a maker of decorative glasses and mirrors in Bavaria, while Fresnel was the son of an architect in Normandy.

What is Diffraction of Light? 

Diffraction is the bending of light around the sharp corner of an obstacle. When light is incident on a slit, with a size comparable to the wavelength of light , an alternating dark and bright pattern can be observed. This phenomenon is called the single slit diffraction. According to Huygens’ principle, when light is incident on the slit, secondary wavelets generate from each point. These wavelets start out in phase and propagate in all directions. Each wavelet travels a different distance to reach any point on the screen. Due to the path difference, they arrive with different phases and interfere constructively or destructively.

Diffraction Due to Single Slit

When light is incident on the sharp edge of an obstacle, a faint illumination can be found within the geometrical shadow of the obstacle. This suggests that light bends around a sharp corner. The effect becomes significant when light passes through an aperture having a dimension comparable to the wavelength of light.

If light is incident on a slit having a width comparable to the wavelength of light, an alternating dark and bright pattern can be seen if a screen is placed in front of the slit. This phenomenon is known as single slit diffraction.

(Image will be uploaded soon)

Young’s Single Slit Experiment

Thomas Young’s double-slit experiment, performed in 1801, demonstrates the wave nature of light. In this experiment, monochromatic light is shone on two narrow slits. The waves, after passing through each slit, superimpose to give an alternate bright and dark distribution on a distant screen. All the bright fringes have the same intensity and width.

In a single slit experiment, monochromatic light is passed through one slit of finite width and a similar pattern is observed on the screen. Unlike the double-slit diffraction pattern, the width and intensity in the single-slit diffraction pattern reduce as we move away from the central maximum.

Explanation of The Phenomenon and Diffraction Formula

According to Huygens’ principle, when light is incident on the slit, secondary wavelets generate from each point. These wavelets start out in phase and propagate in all directions. Each wavelet travels a different distance to reach any point on the screen. Due to the path difference, they arrive with different phases and interfere constructively or destructively.

If a monochromatic light of wavelength λ falls on a slit of width a, the intensity on a screen at a distance L from the slit can be expressed as a function of θ.

Here, θ is the angle made with the original direction of light. It is given by,

I\[\left ( \theta  \right )\] = I\[_{0}\] \[\frac{sin^{2}\alpha }{\alpha ^{2}}\]

Here, \[\alpha\] = \[\frac{\Pi }{\lambda }\] sin\[\theta\]   and I₀ is the intensity of the central bright fringe, located at θ =0.

Diffraction Maxima and Minima 

Bright fringes appear at angles,

\[\theta\] \[\rightarrow\] 0,\[\theta\]\[\rightarrow\]  sin-1 \[\left ( \pm \frac{3\lambda }{2} \right )\], \[\theta\] \[\rightarrow\] sin-1 \[\left ( \pm \frac{5\lambda }{2} \right )\]

\[\theta\] \[\rightarrow\] 0 is the central maximum

Dark fringes correspond to the condition,

a sin\[\theta\] = m\[\lambda\] with m = ±1, ±2, ±3…

In a double slit arrangement, diffraction through single slits appears as an envelope over the interference pattern between the two slits.

Fringe width  

The angular distance between the two first order minima (on either side of the center) is called the angular width of central maximum, given by

2\[\theta\] = \[\frac{2\lambda }{a}\]

The linear width is as follows,

\[\Delta\] = L.2\[\theta\] = \[\frac{2L\lambda }{a}\]

The width of the central maximum in the diffraction formula is inversely proportional to the slit width. If the slit width decreases, the central maximum widens, and if the slit width increases, it narrows down. It can be inferred from this behaviour that light bends more as the dimension of the aperture becomes smaller.

Conditions for Diffraction

The incident light should be monochromatic.

The slit width should be comparable to the wavelength of incident light.

Types of Diffraction

Fresnel Diffraction: The light source and the screen both are at finite distances from the slit. The incident waves are not parallel. 

Fraunhofer Diffraction: The light source and the screen both are infinitely away from the slit such that the incident light rays are parallel.

Solved Examples

1. Fraunhofer diffraction at a single slit is performed using a 700 nm light. If the first dark fringe appears at an angle 30 ° , find the slit width.

Solution: Using the diffraction formula for a single slit of width a, the n th dark fringe occurs for,

a sin\[\theta\] = nλ

At angle θ=30°, the first dark fringe is located. Using n=1 and

λ = 700 nm=700 X 10⁻⁹m,

a sin 30°=1 X 700 X 10⁻⁹m

a=14 X 10⁻⁷m

The slit width is 1400 nm.

2. Find the angular width of central maximum for Fraunhofer diffraction due to a single slit of width 0.1 m, if the frequency of incident light is 5 X 10¹⁴ Hz.

Solution: wavelength of the incident light is,

\[\lambda\] = \[\frac{c}{\nu }\]

Here, c=3 X 10⁸m/s is the speed of light in vacuum and =5 X 10¹⁴Hz  is the frequency.

The angular width of the central maximum is,

2\[\theta\] = \[\frac{2c}{\nu a}\]

Using c = 3×10⁸m/s, = 5×10¹⁴Hz a = 0.1m,

2\[\theta\] = 1.2×10⁻⁴rad

The angular width is 1.210⁻⁴rad

About Fraunhofer and Fresnel

Fraunhofer and Fresnel, both kids faced immense hardships in life, partly because of their weak constitutions. At the end of the first decade of the nineteenth century, the building where Fraunhofer lived collapsed. Fraunhofer was the only one who survived. Maximilian IV Joseph, Prince-Elector of Bavaria, who was busy in the rescue efforts, noticed Fraunhofer and decided to take him under his wing. With the entry of Maximilian into his life, Fraunhofer’s life took a turn for the better.

In the neighbouring country of Italy simultaneously, Fresnel was still struggling to read at the age of eight. Surrounded by his three siblings, Fresnel had toys aplenty. At the age of nine, he managed to transform a harmless toy into such a lethal weapon that elders in his society were forced to ban the use of the now-dangerous toy. This caused no end of worry and consternation to his parents and grown-ups, but his genius was finally recognized. 

He was taught by fine tutors and by the age of 16, he was admitted to the École Polytechnique. Here, an eminent mathematician, Adrien-Marie Legendre noticed Fresnel’s answer paper of striking ingenuity. Fresnel was encouraged in his studies of Maths and Science. He then graduated from École des Ponts et Chaussées as an Engineer.

Fraunhofer, The Scientist

Somewhere down a few longitudes, Maximilian took charge of the education of Fraunhofer and put him under the care of Joseph von Utzschneider, an attorney with a unique business mind. In those days industrious Utzschneider was busy partnering with Georg Reichenbach and J. Leibherr to open the Mathematical Mechanical Institute Reichenbach Utzschneider Liebherr. This institute was established to produce surveying instruments with high-quality lenses – incidentally, just the thing that Fraunhofer had a knack for.

After a failed stint at establishing his own business, Fraunhofer was offered a position in the Institute by Utzschneider. Here, under the guidance of expert glassmaker Pierre Guinand, Fraunhofer shined. Within two years he was offered a junior partnership in the Institute. Within a decade, Fraunhofer had turned the Institute around and was producing quality refractor lenses with a diameter of seven inches. This was an achievement in those times and was ably equipped with a decent-sized telescope.

Fresnel, The Scientist

On the other side, Fresnel was following the footsteps of his father, the architect. Having a few false starts mostly because he supported the monarchy that kept changing at that point in history, Fresnel settled as an engineer for the roads of Paris.

By now the careers of both scientists were converging as both were taking a keen interest in optics. By the middle of the second decade, both Fresnel and Fraunhofer were knee-deep in their research, resulting in scintillating findings that are studied in Optics for class 12 and higher classes.

Fraunhofer had a knack for handling glass and optics. A major challenge at his time was accurately polishing spherical surfaces of large glass objects. With his ingenuity, Fraunhofer improved upon and also invented machines that did more accurate grinding and polishing of glass surfaces. This greatly improved the kinds of lenses being manufactured and thus improved the quality of optical instruments being used.

In 1811, he constructed the furnace that would make the perfect large-sized lenses required in larger telescopes. With this new furnace, Fraunhofer created his own crown glass that had no irregular refractive power as larger crown glasses of that time tended to have.

In 1814, Fraunhofer invented the modern spectroscope through a series of experiments. By observing the orange fixed light in the spectrum of fire, he researched the dark lines of the solar spectrum, as well as in several bright stars, thus initiating research in stellar spectroscopy.

In 1821, he invented the diffraction grating and became the first scientist to get line spectra and then measure the wavelengths of spectral lines. His passion always remained practical optics so he kept developing and bettering optical instruments. He also created the Dorpat Refractor ad and Bessel Heliometer.

The most practical and popular invention of Fresnel was Fresnel Lens. He created this for Napoleon. It is used in theatrical and cinematic lighting fixtures and lighthouses to shine a narrow beam in all directions simultaneously. These are made of a lot of complicated wedge-shaped parts. Fresnel lenses magnify a light beam about 150 times and shine it, thus making it very practical in lighthouses in his time. Today these lenses are popular in film-making and theatre.

Did you know?

In the diffraction pattern of white light , the central maximum is white but the other maxima becomes colored with red being the farthest away.

Diffraction patterns can be obtained for any wave. Subatomic particles like electrons also show similar patterns like light. This observation led to the concept of a particle’s wave nature and it is considered as one of the keystones for the advent of quantum mechanics.

The interatomic distances of certain crystals are comparable with the wavelength of X-rays. Using X-ray diffraction patterns, the crystal structures of different materials are studied in condensed matter physics.

arrow-right

FAQs on Diffraction of Light - Young’s Single Slit Experiment

1. What is Optics?

Optics is a branch of Physics in which the properties of light and how it interacts with other objects, are studied. In everyday life, the uses of Optics can be seen in mirrors, light tunnels, lenses, spectacles, microscopes, and fibre optics.

2. How does Optics connect to Astronomy?

Since travelling enormous distances in Astronomy is not practical, scientists study the light from different astronomical bodies in space to gauge their distance from earth and to each other. Optics helps scientists unravel the secrets of outer space since not much other than light reaches the earth from outer space. The distance in space is measured in Light Years. One light-year is the distance travelled by light in a span of one year.

3. How do we measure the intensity of light?

We can measure how bright a light is. The unit of measuring the luminous intensity of light is Candela. A small candle produces about 1 Cd (Candela) of light. The amount of light falling on a surface is measured in lux. 1 Lu is the amount of light from a 1 Cd strong source 1 meter away. To be able to read one needs 500 lux of light.

4. How do we make the light turn the opposite way from where it is coming?

We can make light change direction by using a mirror. This simple method is used in making light tunnels in newly constructed houses to save energy. A light tunnel captures sunlight and reflects it along a tunnel fitted with mirrors in precise angles so that the other end of the tunnel reflects light inside the house.

5. What is the difference between Fresnel and Fraunhofer class of diffraction?

The light source and the screen both are at finite distances from the slit for Fresnel diffraction whereas the distances are infinite for Fraunhofer diffraction. The incident light rays are parallel (plane wavefront) for the latter. For Fresnel diffraction, the incident light can have a spherical or cylindrical wavefront.

young's single slit experiment class 12

  • Why Does Water Expand When It Freezes
  • Gold Foil Experiment
  • Faraday Cage
  • Oil Drop Experiment
  • Magnetic Monopole
  • Why Do Fireflies Light Up
  • Types of Blood Cells With Their Structure, and Functions
  • The Main Parts of a Plant With Their Functions
  • Parts of a Flower With Their Structure and Functions
  • Parts of a Leaf With Their Structure and Functions
  • Why Does Ice Float on Water
  • Why Does Oil Float on Water
  • How Do Clouds Form
  • What Causes Lightning
  • How are Diamonds Made
  • Types of Meteorites
  • Types of Volcanoes
  • Types of Rocks

Single-Slit Diffraction

Definition: what is the single-slit diffraction.

Single-slit diffraction is the most straightforward experimental setup where diffraction effects can be observed. When light passes through a slit whose width is on the order of the wavelength of light, a distinct diffraction pattern is observed on a screen that is kept at a certain distance from the slit.  The intensity is a function of the angle through which the rays bend.

Huygens’ Principle

According to Huygens’ principle, every unobstructed point on a wavefront will act as a source of secondary spherical waves. The new wavefront is the surface tangent to all the secondary spherical waves. Therefore, each part of the slit can be thought of as an emitter of waves.

young's single slit experiment class 12

Single Slit Diffraction Pattern

All the waves passing through the slit interfere to produce a diffraction pattern consisting of bright and dark fringes. The bright fringes are due to constructive interference, and the dark areas are due to destructive interference. A number represents the order of the bright and dark fringes. The intensities of the fringes consist of a central maximum surrounded by maxima and minima on its either side. The central maximum is brighter than the other maxima. The maxima rapidly decrease as one moves further from the center.

Single Slit Diffraction Equation

In order to study the diffraction pattern on a screen, the single-slit experiment is employed. Consider a monochromatic source of light that passes through a slit AB of width a as shown in the figure. At point P on the screen, the secondary waves interfere destructively and produce a dark fringe. Let D be the distance between the slit and the screen, and y be the distance between point P and point O, the center of the screen. AC is perpendicular to BP. Let θ be the angle of diffraction, and θ’ is the angle BAC.

young's single slit experiment class 12

We assume that the screen is at a considerable distance from the slit, i.e., D >> a . Hence,

θ = θ’

sin θ ≈ tan θ ≈ θ = y/D

The path difference between the two rays AP and BP is given by,

Δ = BP – AP = BC

In the right-angled triangle BCA,

sin θ’ = sin θ = BC/BA

BC = BA sin θ = a sin θ

Δ = a sin θ

Diffraction Minima

The condition for minima or dark fringe is,

Path difference = integral multiple of wavelength

or, Δ = nλ (n=±1, ±2, ±3, … , etc.)

or, a sin θ = nλ

or, ay/D = nλ

or, y n = nλD/a

This equation gives the distance of the n-th dark fringe from the center.

The fringe width is given by,

β  = y n+1 – y n = (n+1)λD/a  –  nλD/a

or, β = λD/a

Diffraction Maxima

The condition for maxima or bright fringe is,

Path difference = non-integral multiple of wavelength

or, Δ = (n+1/2)λ (n=±1, ±2, ±3, … , etc.)

or, a sin θ = (n+1/2)λ

or, ay/D = (n+1/2)λ

or, y n = (n+1/2)λD/a

The intensity of single-slit diffraction is given by,

I = I 0 [sin (π a sin θ/λ)/( π a sin θ/λ)] 2

Difference Between Single- and Double-Slits Diffraction

Consist of one slitConsist of two slits
Waves that originate within the same slit interfereSlits are so small that each one is considered as a single light source, and the interference of waves originating within the same slit can be neglected
The fringes are broad and not as sharp as double-slitThe fringes are narrow and sharp
All bright fringes are visibleSome bright fringes are missing since they suppressed by the minima of a single-slit interference pattern
Intensity:  [sin (π a sin θ/λ)/( π a sin θ/λ)] Intensity:  cos [π d sin θ/λ] [sin (π a sin θ/λ)/( π a sin θ/λ)]
  • Single-Slit Diffraction – Opentextbc.ca
  • Single-Slit Diffraction – Labman.phys.utk.edu
  • Diffraction from a Single Slit – Animations.physics.unsw.edu.au
  • Single-Slit Diffraction – Physics.nus.edu.sg
  • Single-Slit Diffraction – Openstax.org
  • Fraunhofer Single Slit – Hyperphysics.phy-astr.gsu.edu
  • Intensity of Single-Slit Diffraction – Scipp.ucsc.edu
  • Interference and Diffraction – Web.mit.edu

Article was last reviewed on Thursday, February 2, 2023

Related articles

Spectrophotometer

Leave a Reply Cancel reply

Your email address will not be published. Required fields are marked *

Save my name, email, and website in this browser for the next time I comment.

Popular Articles

young's single slit experiment class 12

Join our Newsletter

Fill your E-mail Address

Related Worksheets

  • Privacy Policy

© 2024 ( Science Facts ). All rights reserved. Reproduction in whole or in part without permission is prohibited.

  • Physics Class-11th Notes
  • Physics Formulas
  • Physics Symbol
  • Application of Physics
  • Class 8 Science
  • Class 9 Science
  • Class 10 Science
  • Class 11 Science
  • Class 12 Science
  • Class 8 Study Material
  • Class 9 Study Material
  • Class 10 Study Material
  • Class 11 Study Material
  • Class 12 Study Material

Single Slit Diffraction

Single Slit Diffraction is a fundamental concept in wave optics that explains how light behaves as a wave when passing through a narrow slit. When coherent light (like a laser) goes through a single narrow slit, the waves spread out, and their interaction creates a pattern on a screen placed some distance away. This phenomenon, known as diffraction, leads to the formation of alternating bright and dark regions, showcasing the wave nature of light.

In this article, we’ll learn core concepts, types, and practical applications of Single Slit Diffraction, aiming to simplify and explore its patterns and formulas.

Table of Content

What is Single Slit Diffraction?

Central maximum, path difference, minima position, intensity distribution curve (pattern), single slit diffraction formula.

Diffraction is defined as the phenomenon in which light bends around the corners of an obstacle whose size is comparable to the wavelength of the light

Single Slit Diffraction occurs when light waves pass through a narrow aperture or slit. As the waves spread out, they interfere with each other, resulting in an interference pattern on a screen placed some distance away. This pattern includes a central maximum, secondary maxima, and minima, showing the bending and spreading of light waves around the edges of the slit.

Simulation-of-Single-Slit-Experiment

The central maximum is the brightest region in a single slit diffraction pattern, appearing at the center of the screen. It’s formed due to constructive interference, where light waves traveling straight through the slit reinforce each other, resulting in a more intense central peak.

Central Maximum

The formula for Central Maximum is:

sin θ = nλ/a where, θ is the Angle from the Center to Maxima n is the Order of the Maxima (for central maximum, it is 1) λ is the Wavelength of the incident light a is the Width of the slit

Path difference describes the variation in distances traveled by light waves from different points on the slit to a specific spot on the screen. It plays a critical role in determining how light waves interact—whether constructively or destructively—at that point. Understanding Single Slit Diffraction offers insights into the intriguing behavior of light waves as they pass through small openings, producing distinct patterns that reveal their wave-like nature. The formula for path difference in single slit diffraction is:

L = d⋅sin(θ) where, d represents the width of the slit θ is the angle of diffraction

The spots where we see darkness or dimness between the bright lines in a single slit diffraction pattern are called minima. They happen because some light waves meet at certain places on the screen and mix in a way that makes those spots look darker. This darkness occurs because these mixed light waves cancel each other out, resulting in reduced brightness or darkness in those particular areas.

The formula for Minima is:

a.sin θ= nλ where, θ is the Angle to Minima n is the Order of Minima λ is the Wavelength of the Incident light a is the Width of the Slit

Position of Secondary Maxima

Secondary maxima are smaller and less intense bright areas close to the brightest spot in a single slit diffraction pattern. They appear because some diffracted waves change direction around the edges of the slit, creating slightly brighter regions away from the brightest spot. This happens because these waves add up and reinforce each other at specific angles, making these areas a bit brighter than their surroundings.

In Diffraction by single slit intensity distribution curve shows how the brightness changes across the diffraction pattern. It explains how bright different angles or spots on the screen are. The curve usually has a highest point in the middle (which represents the central maximum) and then the brightness decreases gradually towards the secondary maxima and minima. This means that the brightest point in the middle is usually much brighter than the slightly less bright areas around it.

The diffraction single by single slit can be be best understood by the mathematical formula also called single slit diffraction formula.

The intensity distribution of diffracted light can be expressed by the single slit diffraction formula:

I(θ) = I 0 (sin(β)/ β) 2 where, I(θ) is the intensity at an angle θ I 0 is the intensity at the center of the diffraction pattern β is the phase difference between waves arriving at different points on the screen

Single Slit Diffraction vs Double Slit Diffraction

Diffraction of light can occur through single slit and double slit. However, the pattern observed in different in both the cases. The difference between single slit diffraction and double slit diffraction is tabulated below:

Involves a single narrow slit.

Involves two adjacent slits.

Results in an interference pattern with a central maximum, secondary maxima, and minima.

Displays multiple interference patterns with a series of bright and dark fringes.

Pattern created due to the diffraction of light waves passing through a single slit.

Pattern results from the interference of light waves from both slits.

Shows a central maximum and secondary maxima and minima.

Shows multiple interference patterns that incorporate characteristics of both single-slit and double-slit interference.

Exhibits a central maximum and decreasing intensity on either side with secondary maxima.

Shows alternating bright and dark fringes, with the central maximum being more pronounced and subsequent maxima decreasing in intensity.

Also, Check

Hugyen’s Wave Theory Difference between Diffraction and Interference Young’s Double Slit Diffraction

Single Slit Diffraction Solved Examples

Example 1: A single-slit diffraction experiment uses light of wavelength λ = 600 nm and a slit width of a = 0.1 mm. Calculate the angular position (in degrees) of the first minimum.

a = 0.1 mm = 0.1 × 10 (-3) m λ = 600 nm = 600 × 10 (-9) m For the first minimum, n = 1 Formula for the angular position of the first minimum: sin(θ) = n λ / a Substituting values: sin(θ) = (1) × (600 × 10 (-9) ) / (0.1 × 10 (-3) ) Calculating: sin(θ) ≈ 0.006 Hence, θ ≈ 0.6 degrees

Example 2: In a single-slit diffraction experiment, light of wavelength λ = 500 nm produces the first minimum at an angle x = 0.04 cm from the central maximum. Determine the slit width (in mm).

Given: λ = 500 nm = 500 × 10 -9 m x = 0.04 cm = 0.04 × 10 -2 m For the first minimum, n = 1 Formula for slit width: a = n × λ × x / sin(θ) sin(θ) = x / D (D is the distance between the slit and the screen) Substituting : a = (1) × (500 × 10 (-9) ) × (0.04 × 10 -2 )/ (sin(θ)) Sin(θ) ≈ θ (for small angles), therefore a ≈ 5 × 10 -3 m or 0.05 mm

Example 3. Light of wavelength λ = 500 nm passes through a slit of width a = 0.1 mm, creating a diffraction pattern on a screen. If the first minimum is observed at a distance x = 5 mm from the central maximum, what is the distance (in mm) between the screen and the slit?

Given: λ = 500 nm = 500 × 10 -9 m a = 0.1 mm = 0.1 × 10 -3 m x = 5 mm = 5 × 10 -3 m For the first minimum, n = 1 Formula for distance: D = n λ x / a Substituting values: D = (1) × (500 × 10 -9 ) × (5 × 10 -3 ) / (0.1 × 10 -3 ) D ≈ 6.4 mm

Example 4. For a single-slit diffraction pattern, light of wavelength λ = 500 nm produces the first minimum at an angular separation of Δθ = 60 degrees. Calculate the slit width (in mm).

λ = 500 nm = 500 × 10 -9 m Δθ = 60 degrees For the central maximum, n = 1 Formula for slit width: a = n × λ / sin(Δθ) Substituting values: a = (1) × (500 × 10 -9 ) / sin(60 degrees) Calculating: a ≈ 3.33 × 10 -3 m or 0.033 mm

Example 5: A monochromatic light of wavelength λ = 600 nm passes through a single slit and produces a diffraction pattern on a screen. If the angular width of the central maximum is 10 degrees, determine the width of the slit.

Given: Wavelength (λ) = 600 nm = 600 × 10-9 m Angular width of the central maximum = 10 degrees For the central maximum, n = 1 Formula for the width of the slit: a = n × λ / sin(Δθ) Substituting values: a = (1) × (600 × 10-9) / sin(10 degrees) Calculating: a ≈ 3.47 × 10-5 m or 34.7 μm

Practice Problems on Single Slit Diffraction

Q1: A single-slit diffraction pattern is formed on a screen 2 meters away. If a light of wavelength 500 nm produces the first minimum at an angle of 30 degrees, what is the width of the slit?

Q2: Light with a wavelength of 600 nm passes through a single slit and produces a diffraction pattern. If the angular width of the central maximum is 20 degrees, calculate the width of the slit.

Q3: In a single-slit diffraction experiment, if the second minimum is observed at an angle of 45 degrees and the wavelength of light used is 450 nm, find the width of the slit.

Q4: A diffraction pattern is observed on a screen placed 1.5 meters away from a single slit. If light of wavelength 700 nm produces the first minimum at an angle of 45 degrees, determine the width of the slit.

Q5: When monochromatic light with a wavelength of 400 nm passes through a single slit, the first minimum is observed at an angle of 15 degrees. What is the width of the slit?

Single Slit Diffraction – FAQs

1. what is diffraction.

The bending of light around the corners of obstacle whose size is comparable to the wavelength of the light is called Diffraction

2. What causes Single Slit Diffraction?

Single Slit Diffraction is primarily caused by the wave nature of light. As light waves pass through a single slit, they interfere with each other, leading to the formation of a distinct diffraction pattern.

2. How does the Diffraction Pattern change with different Slit Widths?

The width of the slit directly influences the spacing and intensity of the diffraction maxima and minima. Wider slits result in broader and less intense patterns, while narrower slits yield sharper and more concentrated patterns.

4. Can Single Slit Diffraction occur with other forms of Waves?

Single Slit Diffraction is a universal wave phenomenon, and it can manifest with various types of waves, not limited to light waves.

5. What is the Difference between Fraunhofer and Fresnel Diffraction?

The key distinction lies in the distances involved. Fraunhofer Diffraction assumes an infinitely distant light source, whereas Fresnel Diffraction considers finite distances between the source, slit, and observing screen. This difference in perspective leads to variations in the observed diffraction patterns.

6. How does changing the Wavelength affect the Diffraction Pattern?

The wavelength of light directly influences the spacing of the diffraction maxima and minima. Shorter wavelengths result in narrower patterns, impacting the overall appearance of the diffraction pattern on the screen

7. What is Diffraction from One Slit?

Diffraction from one slit refers to the bending and spreading of light waves when they pass through a single narrow aperture or slit, causing the waves to spread out and interfere with each other, forming an interference pattern on a screen.

8. What are the two main Types of Diffraction?

The two main types of diffraction are: Single Slit Diffraction: Occurs when light passes through a single narrow aperture, resulting in an interference pattern with a central maximum and secondary maxima. Double Slit Diffraction: Involves two adjacent slits, producing multiple interference patterns on a screen, displaying a series of bright and dark fringes.

Please Login to comment...

Similar reads.

  • Geeks Premier League
  • School Learning
  • School Physics
  • Geeks Premier League 2023
  • Physics-Class-11
  • Wave-Optics
  • Discord Emojis List 2024: Copy and Paste
  • Best Adblockers for Twitch TV: Enjoy Ad-Free Streaming in 2024
  • PS4 vs. PS5: Which PlayStation Should You Buy in 2024?
  • Best Mobile Game Controllers in 2024: Top Picks for iPhone and Android
  • System Design Netflix | A Complete Architecture

Improve your Coding Skills with Practice

 alt=

What kind of Experience do you want to share?

Youtube

  • TPC and eLearning
  • What's NEW at TPC?
  • Read Watch Interact
  • Practice Review Test
  • Teacher-Tools
  • Request a Demo
  • Get A Quote
  • Subscription Selection
  • Seat Calculator
  • Ad Free Account
  • Edit Profile Settings
  • Metric Conversions Questions
  • Metric System Questions
  • Metric Estimation Questions
  • Significant Digits Questions
  • Proportional Reasoning
  • Acceleration
  • Distance-Displacement
  • Dots and Graphs
  • Graph That Motion
  • Match That Graph
  • Name That Motion
  • Motion Diagrams
  • Pos'n Time Graphs Numerical
  • Pos'n Time Graphs Conceptual
  • Up And Down - Questions
  • Balanced vs. Unbalanced Forces
  • Change of State
  • Force and Motion
  • Mass and Weight
  • Match That Free-Body Diagram
  • Net Force (and Acceleration) Ranking Tasks
  • Newton's Second Law
  • Normal Force Card Sort
  • Recognizing Forces
  • Air Resistance and Skydiving
  • Solve It! with Newton's Second Law
  • Which One Doesn't Belong?
  • Component Addition Questions
  • Head-to-Tail Vector Addition
  • Projectile Mathematics
  • Trajectory - Angle Launched Projectiles
  • Trajectory - Horizontally Launched Projectiles
  • Vector Addition
  • Vector Direction
  • Which One Doesn't Belong? Projectile Motion
  • Forces in 2-Dimensions
  • Being Impulsive About Momentum
  • Explosions - Law Breakers
  • Hit and Stick Collisions - Law Breakers
  • Case Studies: Impulse and Force
  • Impulse-Momentum Change Table
  • Keeping Track of Momentum - Hit and Stick
  • Keeping Track of Momentum - Hit and Bounce
  • What's Up (and Down) with KE and PE?
  • Energy Conservation Questions
  • Energy Dissipation Questions
  • Energy Ranking Tasks
  • LOL Charts (a.k.a., Energy Bar Charts)
  • Match That Bar Chart
  • Words and Charts Questions
  • Name That Energy
  • Stepping Up with PE and KE Questions
  • Case Studies - Circular Motion
  • Circular Logic
  • Forces and Free-Body Diagrams in Circular Motion
  • Gravitational Field Strength
  • Universal Gravitation
  • Angular Position and Displacement
  • Linear and Angular Velocity
  • Angular Acceleration
  • Rotational Inertia
  • Balanced vs. Unbalanced Torques
  • Getting a Handle on Torque
  • Torque-ing About Rotation
  • Properties of Matter
  • Fluid Pressure
  • Buoyant Force
  • Sinking, Floating, and Hanging
  • Pascal's Principle
  • Flow Velocity
  • Bernoulli's Principle
  • Balloon Interactions
  • Charge and Charging
  • Charge Interactions
  • Charging by Induction
  • Conductors and Insulators
  • Coulombs Law
  • Electric Field
  • Electric Field Intensity
  • Polarization
  • Case Studies: Electric Power
  • Know Your Potential
  • Light Bulb Anatomy
  • I = ∆V/R Equations as a Guide to Thinking
  • Parallel Circuits - ∆V = I•R Calculations
  • Resistance Ranking Tasks
  • Series Circuits - ∆V = I•R Calculations
  • Series vs. Parallel Circuits
  • Equivalent Resistance
  • Period and Frequency of a Pendulum
  • Pendulum Motion: Velocity and Force
  • Energy of a Pendulum
  • Period and Frequency of a Mass on a Spring
  • Horizontal Springs: Velocity and Force
  • Vertical Springs: Velocity and Force
  • Energy of a Mass on a Spring
  • Decibel Scale
  • Frequency and Period
  • Closed-End Air Columns
  • Name That Harmonic: Strings
  • Rocking the Boat
  • Wave Basics
  • Matching Pairs: Wave Characteristics
  • Wave Interference
  • Waves - Case Studies
  • Color Addition and Subtraction
  • Color Filters
  • If This, Then That: Color Subtraction
  • Light Intensity
  • Color Pigments
  • Converging Lenses
  • Curved Mirror Images
  • Law of Reflection
  • Refraction and Lenses
  • Total Internal Reflection
  • Who Can See Who?
  • Lab Equipment
  • Lab Procedures
  • Formulas and Atom Counting
  • Atomic Models
  • Bond Polarity
  • Entropy Questions
  • Cell Voltage Questions
  • Heat of Formation Questions
  • Reduction Potential Questions
  • Oxidation States Questions
  • Measuring the Quantity of Heat
  • Hess's Law
  • Oxidation-Reduction Questions
  • Galvanic Cells Questions
  • Thermal Stoichiometry
  • Molecular Polarity
  • Quantum Mechanics
  • Balancing Chemical Equations
  • Bronsted-Lowry Model of Acids and Bases
  • Classification of Matter
  • Collision Model of Reaction Rates
  • Density Ranking Tasks
  • Dissociation Reactions
  • Complete Electron Configurations
  • Elemental Measures
  • Enthalpy Change Questions
  • Equilibrium Concept
  • Equilibrium Constant Expression
  • Equilibrium Calculations - Questions
  • Equilibrium ICE Table
  • Intermolecular Forces Questions
  • Ionic Bonding
  • Lewis Electron Dot Structures
  • Limiting Reactants
  • Line Spectra Questions
  • Mass Stoichiometry
  • Measurement and Numbers
  • Metals, Nonmetals, and Metalloids
  • Metric Estimations
  • Metric System
  • Molarity Ranking Tasks
  • Mole Conversions
  • Name That Element
  • Names to Formulas
  • Names to Formulas 2
  • Nuclear Decay
  • Particles, Words, and Formulas
  • Periodic Trends
  • Precipitation Reactions and Net Ionic Equations
  • Pressure Concepts
  • Pressure-Temperature Gas Law
  • Pressure-Volume Gas Law
  • Chemical Reaction Types
  • Significant Digits and Measurement
  • States Of Matter Exercise
  • Stoichiometry Law Breakers
  • Stoichiometry - Math Relationships
  • Subatomic Particles
  • Spontaneity and Driving Forces
  • Gibbs Free Energy
  • Volume-Temperature Gas Law
  • Acid-Base Properties
  • Energy and Chemical Reactions
  • Chemical and Physical Properties
  • Valence Shell Electron Pair Repulsion Theory
  • Writing Balanced Chemical Equations
  • Mission CG1
  • Mission CG10
  • Mission CG2
  • Mission CG3
  • Mission CG4
  • Mission CG5
  • Mission CG6
  • Mission CG7
  • Mission CG8
  • Mission CG9
  • Mission EC1
  • Mission EC10
  • Mission EC11
  • Mission EC12
  • Mission EC2
  • Mission EC3
  • Mission EC4
  • Mission EC5
  • Mission EC6
  • Mission EC7
  • Mission EC8
  • Mission EC9
  • Mission RL1
  • Mission RL2
  • Mission RL3
  • Mission RL4
  • Mission RL5
  • Mission RL6
  • Mission KG7
  • Mission RL8
  • Mission KG9
  • Mission RL10
  • Mission RL11
  • Mission RM1
  • Mission RM2
  • Mission RM3
  • Mission RM4
  • Mission RM5
  • Mission RM6
  • Mission RM8
  • Mission RM10
  • Mission LC1
  • Mission RM11
  • Mission LC2
  • Mission LC3
  • Mission LC4
  • Mission LC5
  • Mission LC6
  • Mission LC8
  • Mission SM1
  • Mission SM2
  • Mission SM3
  • Mission SM4
  • Mission SM5
  • Mission SM6
  • Mission SM8
  • Mission SM10
  • Mission KG10
  • Mission SM11
  • Mission KG2
  • Mission KG3
  • Mission KG4
  • Mission KG5
  • Mission KG6
  • Mission KG8
  • Mission KG11
  • Mission F2D1
  • Mission F2D2
  • Mission F2D3
  • Mission F2D4
  • Mission F2D5
  • Mission F2D6
  • Mission KC1
  • Mission KC2
  • Mission KC3
  • Mission KC4
  • Mission KC5
  • Mission KC6
  • Mission KC7
  • Mission KC8
  • Mission AAA
  • Mission SM9
  • Mission LC7
  • Mission LC9
  • Mission NL1
  • Mission NL2
  • Mission NL3
  • Mission NL4
  • Mission NL5
  • Mission NL6
  • Mission NL7
  • Mission NL8
  • Mission NL9
  • Mission NL10
  • Mission NL11
  • Mission NL12
  • Mission MC1
  • Mission MC10
  • Mission MC2
  • Mission MC3
  • Mission MC4
  • Mission MC5
  • Mission MC6
  • Mission MC7
  • Mission MC8
  • Mission MC9
  • Mission RM7
  • Mission RM9
  • Mission RL7
  • Mission RL9
  • Mission SM7
  • Mission SE1
  • Mission SE10
  • Mission SE11
  • Mission SE12
  • Mission SE2
  • Mission SE3
  • Mission SE4
  • Mission SE5
  • Mission SE6
  • Mission SE7
  • Mission SE8
  • Mission SE9
  • Mission VP1
  • Mission VP10
  • Mission VP2
  • Mission VP3
  • Mission VP4
  • Mission VP5
  • Mission VP6
  • Mission VP7
  • Mission VP8
  • Mission VP9
  • Mission WM1
  • Mission WM2
  • Mission WM3
  • Mission WM4
  • Mission WM5
  • Mission WM6
  • Mission WM7
  • Mission WM8
  • Mission WE1
  • Mission WE10
  • Mission WE2
  • Mission WE3
  • Mission WE4
  • Mission WE5
  • Mission WE6
  • Mission WE7
  • Mission WE8
  • Mission WE9
  • Vector Walk Interactive
  • Name That Motion Interactive
  • Kinematic Graphing 1 Concept Checker
  • Kinematic Graphing 2 Concept Checker
  • Graph That Motion Interactive
  • Two Stage Rocket Interactive
  • Rocket Sled Concept Checker
  • Force Concept Checker
  • Free-Body Diagrams Concept Checker
  • Free-Body Diagrams The Sequel Concept Checker
  • Skydiving Concept Checker
  • Elevator Ride Concept Checker
  • Vector Addition Concept Checker
  • Vector Walk in Two Dimensions Interactive
  • Name That Vector Interactive
  • River Boat Simulator Concept Checker
  • Projectile Simulator 2 Concept Checker
  • Projectile Simulator 3 Concept Checker
  • Hit the Target Interactive
  • Turd the Target 1 Interactive
  • Turd the Target 2 Interactive
  • Balance It Interactive
  • Go For The Gold Interactive
  • Egg Drop Concept Checker
  • Fish Catch Concept Checker
  • Exploding Carts Concept Checker
  • Collision Carts - Inelastic Collisions Concept Checker
  • Its All Uphill Concept Checker
  • Stopping Distance Concept Checker
  • Chart That Motion Interactive
  • Roller Coaster Model Concept Checker
  • Uniform Circular Motion Concept Checker
  • Horizontal Circle Simulation Concept Checker
  • Vertical Circle Simulation Concept Checker
  • Race Track Concept Checker
  • Gravitational Fields Concept Checker
  • Orbital Motion Concept Checker
  • Angular Acceleration Concept Checker
  • Balance Beam Concept Checker
  • Torque Balancer Concept Checker
  • Aluminum Can Polarization Concept Checker
  • Charging Concept Checker
  • Name That Charge Simulation
  • Coulomb's Law Concept Checker
  • Electric Field Lines Concept Checker
  • Put the Charge in the Goal Concept Checker
  • Circuit Builder Concept Checker (Series Circuits)
  • Circuit Builder Concept Checker (Parallel Circuits)
  • Circuit Builder Concept Checker (∆V-I-R)
  • Circuit Builder Concept Checker (Voltage Drop)
  • Equivalent Resistance Interactive
  • Pendulum Motion Simulation Concept Checker
  • Mass on a Spring Simulation Concept Checker
  • Particle Wave Simulation Concept Checker
  • Boundary Behavior Simulation Concept Checker
  • Slinky Wave Simulator Concept Checker
  • Simple Wave Simulator Concept Checker
  • Wave Addition Simulation Concept Checker
  • Standing Wave Maker Simulation Concept Checker
  • Color Addition Concept Checker
  • Painting With CMY Concept Checker
  • Stage Lighting Concept Checker
  • Filtering Away Concept Checker
  • InterferencePatterns Concept Checker
  • Young's Experiment Interactive
  • Plane Mirror Images Interactive
  • Who Can See Who Concept Checker
  • Optics Bench (Mirrors) Concept Checker
  • Name That Image (Mirrors) Interactive
  • Refraction Concept Checker
  • Total Internal Reflection Concept Checker
  • Optics Bench (Lenses) Concept Checker
  • Kinematics Preview
  • Velocity Time Graphs Preview
  • Moving Cart on an Inclined Plane Preview
  • Stopping Distance Preview
  • Cart, Bricks, and Bands Preview
  • Fan Cart Study Preview
  • Friction Preview
  • Coffee Filter Lab Preview
  • Friction, Speed, and Stopping Distance Preview
  • Up and Down Preview
  • Projectile Range Preview
  • Ballistics Preview
  • Juggling Preview
  • Marshmallow Launcher Preview
  • Air Bag Safety Preview
  • Colliding Carts Preview
  • Collisions Preview
  • Engineering Safer Helmets Preview
  • Push the Plow Preview
  • Its All Uphill Preview
  • Energy on an Incline Preview
  • Modeling Roller Coasters Preview
  • Hot Wheels Stopping Distance Preview
  • Ball Bat Collision Preview
  • Energy in Fields Preview
  • Weightlessness Training Preview
  • Roller Coaster Loops Preview
  • Universal Gravitation Preview
  • Keplers Laws Preview
  • Kepler's Third Law Preview
  • Charge Interactions Preview
  • Sticky Tape Experiments Preview
  • Wire Gauge Preview
  • Voltage, Current, and Resistance Preview
  • Light Bulb Resistance Preview
  • Series and Parallel Circuits Preview
  • Thermal Equilibrium Preview
  • Linear Expansion Preview
  • Heating Curves Preview
  • Electricity and Magnetism - Part 1 Preview
  • Electricity and Magnetism - Part 2 Preview
  • Vibrating Mass on a Spring Preview
  • Period of a Pendulum Preview
  • Wave Speed Preview
  • Slinky-Experiments Preview
  • Standing Waves in a Rope Preview
  • Sound as a Pressure Wave Preview
  • DeciBel Scale Preview
  • DeciBels, Phons, and Sones Preview
  • Sound of Music Preview
  • Shedding Light on Light Bulbs Preview
  • Models of Light Preview
  • Electromagnetic Radiation Preview
  • Electromagnetic Spectrum Preview
  • EM Wave Communication Preview
  • Digitized Data Preview
  • Light Intensity Preview
  • Concave Mirrors Preview
  • Object Image Relations Preview
  • Snells Law Preview
  • Reflection vs. Transmission Preview
  • Magnification Lab Preview
  • Reactivity Preview
  • Ions and the Periodic Table Preview
  • Periodic Trends Preview
  • Chemical Reactions Preview
  • Intermolecular Forces Preview
  • Melting Points and Boiling Points Preview
  • Bond Energy and Reactions Preview
  • Reaction Rates Preview
  • Ammonia Factory Preview
  • Stoichiometry Preview
  • Nuclear Chemistry Preview
  • Gaining Teacher Access
  • Task Tracker Directions
  • Conceptual Physics Course
  • On-Level Physics Course
  • Honors Physics Course
  • Chemistry Concept Builders
  • All Chemistry Resources
  • Users Voice
  • Tasks and Classes
  • Webinars and Trainings
  • Subscription
  • Subscription Locator
  • 1-D Kinematics
  • Newton's Laws
  • Vectors - Motion and Forces in Two Dimensions
  • Momentum and Its Conservation
  • Work and Energy
  • Circular Motion and Satellite Motion
  • Thermal Physics
  • Static Electricity
  • Electric Circuits
  • Vibrations and Waves
  • Sound Waves and Music
  • Light and Color
  • Reflection and Mirrors
  • Measurement and Calculations
  • About the Physics Interactives
  • Task Tracker
  • Usage Policy
  • Newtons Laws
  • Vectors and Projectiles
  • Forces in 2D
  • Momentum and Collisions
  • Circular and Satellite Motion
  • Balance and Rotation
  • Electromagnetism
  • Waves and Sound
  • Atomic Physics
  • Forces in Two Dimensions
  • Work, Energy, and Power
  • Circular Motion and Gravitation
  • Sound Waves
  • 1-Dimensional Kinematics
  • Circular, Satellite, and Rotational Motion
  • Einstein's Theory of Special Relativity
  • Waves, Sound and Light
  • QuickTime Movies
  • About the Concept Builders
  • Pricing For Schools
  • Directions for Version 2
  • Measurement and Units
  • Relationships and Graphs
  • Rotation and Balance
  • Vibrational Motion
  • Reflection and Refraction
  • Teacher Accounts
  • Kinematic Concepts
  • Kinematic Graphing
  • Wave Motion
  • Sound and Music
  • About CalcPad
  • 1D Kinematics
  • Vectors and Forces in 2D
  • Simple Harmonic Motion
  • Rotational Kinematics
  • Rotation and Torque
  • Rotational Dynamics
  • Electric Fields, Potential, and Capacitance
  • Transient RC Circuits
  • Light Waves
  • Units and Measurement
  • Stoichiometry
  • Molarity and Solutions
  • Thermal Chemistry
  • Acids and Bases
  • Kinetics and Equilibrium
  • Solution Equilibria
  • Oxidation-Reduction
  • Nuclear Chemistry
  • Newton's Laws of Motion
  • Work and Energy Packet
  • Static Electricity Review
  • NGSS Alignments
  • 1D-Kinematics
  • Projectiles
  • Circular Motion
  • Magnetism and Electromagnetism
  • Graphing Practice
  • About the ACT
  • ACT Preparation
  • For Teachers
  • Other Resources
  • Solutions Guide
  • Solutions Guide Digital Download
  • Motion in One Dimension
  • Work, Energy and Power
  • Chemistry of Matter
  • Names and Formulas
  • Algebra Based On-Level Physics
  • Honors Physics
  • Conceptual Physics
  • Other Tools
  • Frequently Asked Questions
  • Purchasing the Download
  • Purchasing the Digital Download
  • About the NGSS Corner
  • NGSS Search
  • Force and Motion DCIs - High School
  • Energy DCIs - High School
  • Wave Applications DCIs - High School
  • Force and Motion PEs - High School
  • Energy PEs - High School
  • Wave Applications PEs - High School
  • Crosscutting Concepts
  • The Practices
  • Physics Topics
  • NGSS Corner: Activity List
  • NGSS Corner: Infographics
  • About the Toolkits
  • Position-Velocity-Acceleration
  • Position-Time Graphs
  • Velocity-Time Graphs
  • Newton's First Law
  • Newton's Second Law
  • Newton's Third Law
  • Terminal Velocity
  • Projectile Motion
  • Forces in 2 Dimensions
  • Impulse and Momentum Change
  • Momentum Conservation
  • Work-Energy Fundamentals
  • Work-Energy Relationship
  • Roller Coaster Physics
  • Satellite Motion
  • Electric Fields
  • Circuit Concepts
  • Series Circuits
  • Parallel Circuits
  • Describing-Waves
  • Wave Behavior Toolkit
  • Standing Wave Patterns
  • Resonating Air Columns
  • Wave Model of Light
  • Plane Mirrors
  • Curved Mirrors
  • Teacher Guide
  • Using Lab Notebooks
  • Current Electricity
  • Light Waves and Color
  • Reflection and Ray Model of Light
  • Refraction and Ray Model of Light
  • Teacher Resources
  • Subscriptions

young's single slit experiment class 12

  • Newton's Laws
  • Einstein's Theory of Special Relativity
  • About Concept Checkers
  • School Pricing
  • Newton's Laws of Motion
  • Newton's First Law
  • Newton's Third Law

Young's Experiment

  • Anatomy of a Two-Point Source Interference Pattern
  • The Path Difference
  • Young's Equation
  • Young's Experiment
  • Other Applications of Two-Point Source Interference

young's single slit experiment class 12

Today's classroom version of the same experiment is typically performed using a laser beam as the source. Rather than using a note card to split the single beam into two coherent beams, a carbon-coated glass slide with two closely spaced etched slits is used. The slide with its slits is most commonly purchased from a manufacturer who provides a measured value for the slit separation distance - the d value in Young's equation. Light from the laser beam diffracts through the slits and emerges as two separate coherent waves. The interference pattern is then projected onto a screen where reliable measurements can be made of L and y for a given bright spot with order value m . Knowing these four values allows a student to determine the value of the wavelength of the original light source.

To illustrate some typical results from this experiment and the subsequent analysis, consider the sample data provided below for d, y, L and m.

)

)

to AN ( )

)

(Note: AN 0 = central antinode and AN 4 = fourth antinode)  

The determination of the wavelength demands that the above values for d, y, L and m be substituted into Young's equation.

Careful inspection of the units of measurement is always advisable. The sample data here reveal that each measured quantity is recorded with a different unit. Before substituting these measured values into the above equation, it is important to give some thought to the treatment of units. One means of resolving the issue of nonuniform units is to simply pick a unit of length and to convert all quantities to that unit. If doing so, one might want to pick a unit that one of the data values already has so that there is one less conversion. A wise choice is to choose the meter as the unit to which all other measured values are converted. Since there are 1000 millimeters in 1 meter, the 0.250 mm is equivalent to 0.000250 meter. And since there are 100 centimeters in 1 meter, the 10.2 cm is equivalent to 0.102 m. Thus, the new values of d, y and L are:

While the conversion of all the data to the same unit is not the only means of treating such measured values, it might be the most advisable - particularly for those students who are less at ease with such conversions.

Now that the issue regarding the units of measurement has been resolved, substitution of the measured values into Young's equation can be performed.

λ  = 6.52 x 10 -7 m

As is evident here, the wavelength of visible light is rather small. For this reason wavelength is often expressed using the unit nanometer, where 1 meter is equivalent to 10 9 nanometers. Multiplying by 10 9 will convert the wavelength from meters to nanometers (abbreviated nm).

We Would Like to Suggest ...

young's single slit experiment class 12

Check Your Understanding

1. The diagram below depicts the results of Young's Experiment. The appropriate measurements are listed on the diagram. Use these measurements to determine the wavelength of light in nanometers. (GIVEN: 1 meter = 10 9 nanometers)

Answer: 657 nm

First, identify known values in terms of their corresponding variable symbol:

L = 10.2 m = 1020 cm y = 22.5 cm m = 10 d = 0.298 mm = 0.0298 cm

(Note: m was chosen as 10 since the y distance corresponds to the distance from the 5th bright band on one side of the central band and the 5th bright band on the other side of the central band.)

Then convert all known values to an identical unit. In this case, cm has been chosen as the unit to use. The converted values are listed in the table above.

Substitute all values into Young's equation and perform calculation of the wavelength. The unit of wavelength is cm.

λ = y • d / ( m • L) λ = ( 22.5 cm ) • ( 0.0298 cm ) / [ ( 10 ) • ( 1020 cm ) ] λ = 6.57 x 10 -5 cm

Finally convert to nanometers using a conversion factor. If there are 10 9 nm in 1 meter, then there must be 10 7 nm in the smaller centimeter.

λ = ( 6.57 x 10 -5 cm ) • ( 10 7 nm / 1 cm ) = 657 nm

2. A student uses a laser and a double-slit apparatus to project a two-point source light interference pattern onto a whiteboard located 5.87 meters away. The distance measured between the central bright band and the fourth bright band is 8.21 cm. The slits are separated by a distance of 0.150 mm. What would be the measured wavelength of light?

Answer: 524 nm

L = 5.87 m = 587 cm y = 8.21 cm m = 4 d = 0.150 mm = 0.0150 cm
λ = y • d / ( m • L) λ = ( 8.21 cm ) • ( 0.0150 cm ) / [ ( 4 ) • ( 587 cm ) ] λ = 5.24 x 10 -5 cm
λ = ( 5.24 x 10 -5 cm ) • ( 10 7 nm / 1 cm ) = 524 nm

3. The analysis of any two-point source interference pattern and a successful determination of wavelength demands an ability to sort through the measured information and equating the values with the symbols in Young's equation. Apply your understanding by interpreting the following statements and identifying the values of y, d, m and L. Finally, perform some conversions of the given information such that all information share the same unit.

y =

d =

m =

L =

This question simply asks to equate the stated information with the variables of Young's equation and to perform conversions such that all information is in the same unit.

y = 12.8 cm

d = 0.250 mm

m = 4.5

L = 8.2 meters

y = 12.8 cm

d = 0.0250 cm

m = 4.5

L = 820 cm

(Note that m = 4.5 represents the fifth nodal position or dark band from the central bright band. Also note that the given values have been converted to cm.)

b. An interference pattern is produced when light is incident upon two slits that are 50.0 micrometers apart. The perpendicular distance from the midpoint between the slits to the screen is 7.65 m. The distance between the two third-order antinodes on opposite sides of the pattern is 32.9 cm.

This question simply asks to equate the stated information with the variables of Young's equation and to perform conversions such that all information is in the same unit. 

y = 32.9 cm

d = 50.0 µm

m = 6

L = 7.65 m

y = 32.9 cm

d = 0.00500 cm

m = 6

L = 765 cm

(Note that m = 6 corresponds to six spacings. There are three spacings between the central antinode and the third antinode. The stated distance is twice as far so the m value must be doubled. Also note that the given values have been converted to cm. There are 10 6 µm in one meter; so there are 10 4 µm in one centimeter.)

c. The fourth nodal line on an interference pattern is 8.4 cm from the first antinodal line when the screen is placed 235 cm from the slits. The slits are separated by 0.25 mm.

y = 8.4 cm

d = 0.25 mm

m = 2.5

L = 235 cm

y = 8.4 cm

d = 0.025 cm

m = 2.5

L = 235 cm

 ( Note that the fourth nodal line is assigned the order value of 3.5. Also note that the given values have been converted to cm.)

d. Two sources separated by 0.500 mm produce an interference pattern 525 cm away. The fifth and the second antinodal line on the same side of the pattern are separated by 98 mm.

y = 98 mm

d = 0.500 mm

m = 3

L = 525 cm

y = 9.8 cm

d = 0.0500 cm

m = 3

L = 525 cm

  ( Note that there are three spacings between the second and the fifth bright bands. Since all spacings are the same distance apart, the distance between the second and the fifth bright bands would be the same as the distance between the central and the third bright bands. Thus, m = 3. Also note that the given values have been converted to cm.)

e. Two slits that are 0.200 mm apart produce an interference pattern on a screen such that the central maximum and the 10th bright band are distanced by an amount equal to one-tenth the distance from the slits to the screen.

y = 0.1 • L

d = 0.200 mm

m = 10

L - not stated

y = 0.1 • L

d = 0.200 mm

m = 10

L - not stated

 ( Note that there are 10 spacings between the central anti-node and the tenth bright band or tenth anti-node. And observe that they do not state the actual values of L and y; the value of y is expressed in terms of L. )

f. The fifth antinodal line and the second nodal line on the opposite side of an interference pattern are separated by a distance of 32.1 cm when the slits are 6.5 m from the screen. The slits are separated by 25.0 micrometers.

y = 32.1 cm

d = 25.0 µm

m = 6.5

L = 6.5 m

y = 32.1 cm

d = 0.00250 cm

m = 6.5

L = 650 cm

 ( Note that there are five spacings between the central anti-node and the fifth anti-node. And there are 1.5 spacings from the central anti-node in the opposite direction out to the second nodal line. Thus, m = 6.5. Also note that the given values have been converted to cm. There are 10 6 µm in one meter; so there are 10 4 µm in one centimeter.)

g. If two slits 0.100 mm apart are separated from a screen by a distance of 300 mm, then the first-order minimum will be 1 cm from the central maximum.

y = 1 cm

d = 0.100 mm

m = 0.5

L = 300 mm

y = 1 cm

d = 0.0100 cm

m = 0.5

L = 30.0 cm

  ( Note that a the first-order minimum is a point of minimum brightness or a nodal position. The first-order minimum is the first nodal position and is thus the m = 0.5 node. Also note that the given values have been converted to cm. )

h. Consecutive bright bands on an interference pattern are 3.5 cm apart when the slide containing the slits is 10.0 m from the screen. The slit separation distance is 0.050 mm.

y = 3.5 cm

d = 0.050 mm

m = 1

L = 10.0 m

y = 3.5 cm

d = 0.0050 cm

m = 1

L = 1000 cm

   ( Note that the spacing between adjacent bands is given. This distance is equivalent with the distance from the central bright band to the first antinode. Thus, m = 1. Also note that the given values have been converted to cm. )

cd_logo

  • Study Abroad Get upto 50% discount on Visa Fees
  • Top Universities & Colleges
  • Abroad Exams
  • Top Courses
  • Read College Reviews
  • Admission Alerts 2024
  • Education Loan
  • Institute (Counselling, Coaching and More)
  • Ask a Question
  • College Predictor
  • Test Series
  • Practice Questions
  • Course Finder
  • Scholarship
  • All Courses
  • B.Sc (Nursing)

Single Slit Diffraction: Young’s Single Slit Experiment and Fringe Width

young's single slit experiment class 12

Jasmine Grover

Content Strategy Manager

Single slit diffraction takes place when light is incident on a slit with a size comparable to the wavelength of light and an alternating dark and bright pattern can be observed.  Diffraction implies the bending of light around the sharp corner of an obstacle.   Diffraction can be seen when the sources are small enough that they are fairly the size of light’s wavelength. 

Key Terms:  Diffraction, Single Slit Diffraction, Young’s Single Slit Experiment, Fringe Width,  Huygens' Guideline , Wavelength, Light, Wave, Obstacles

What is Diffraction?

[Click Here for Previous Year Questions]

Diffraction is the inclination of a wave produced from a limited source or passing through a limited gap to spread out around obstacles . Diffraction results from the impedance of an endless number of waves transmitted by constant circulation of source focus in a few measurements.

Diffraction

For example, the silver lining occurring in the sky is caused by the diffraction of light. Thus, Diffraction is the slight bending of light as it passes around the edge of an object. The measure of the bending of light relies upon the overall size of the frequency of light to the size of the opening. However, if the opening is bigger than the light's frequency, the bend might be unnoticeable. 

Relatable Links:

What is Single Slit Diffraction?

[Click Here for Sample Questions]

Single-slit diffraction can be defined as an observation where the bending of light (or, diffraction) causes the coherent source of light to interfere among themselves in order to produce a certain pattern on the screen, known as the Diffraction Pattern.

When the light goes through a single slit with a width w ( frequency of the light), then a single slit diffraction forms on a screen with distance L >> w away from the slit. Huygens' guideline reveals that each piece of the slit can be considered a producer of waves. This load of waves meddles to deliver the diffraction pattern. 

Single Slit Diffraction

Single Slit Diffraction

In other words, if the light is incident on a slit with a width that is comparable to the wavelength of light, then an alternating shadow and bright pattern forms on a screen erected in front of the slit. This phenomenon of light is known as Single Slit Diffraction.

Young's Single Slit Experiment

According to Thomas Young’s double-slit experiment , performed in 1801, the wave nature of light was demonstrated. In this experiment, monochromatic light was illustrated on two narrow slits. After passing through each slit, the waves superimpose to give an alternate bright and dark distribution on a distant screen. And every bright fringe has the same intensity and width. 

Young's Single Slit Experiment

Monochromatic light in a single-slit experiment is transferred through one slit of finite width and an identical pattern is observed on the screen. As we move away from the central maximum, unlike the double-slit diffraction pattern, the width and intensity in the single-slit diffraction pattern reduce.

Single Slit Diffraction Formula

In the event that a monochromatic light of frequency λ falls on a slit of width a, the force on a screen a ways off L from the slit can be communicated as a component of θ. Here, θ is the point made with the first course of light. It is given by,

α/α  … (1)

Here, α = π*λ

Sin θ and I0 are the intensity of the central bright fringe, situated at θ = 0.

Single Slit Diffraction Formula

Diffraction Maxima and Minima: Bright edges show up at points,

θ = 0, θ = sin-1 (±3λ/2), θ = sin-1 (±5λ/2) 

θ = 0 is the focal most extreme.

   … (2)

Dark fringes compared to the condition,

(3)

In a double-slit arrangement, diffraction through a single slit shows up as an envelope over the obstruction design between the two slits.

Fringe Width

The angular distance between the two first-order minima (on one or the other side of the centre) is known as the angular width of the central maximum, given by:

The straight width is as per the following,

Δ = L \(\times\) 2θ= 2Lλa … (4)

The width of the central maximum in the diffraction formula is in inverse proportion with the slit width. Therefore, if the slit width decreases, the central maximum widens, and if the slit width increases, it narrows down. Hence, it can be concluded from this behaviour that light bends more as the dimension of the aperture becomes smaller.

Central Maximum

The maxima placed between the minima and the width of the central maximum can be expressed as the distance between the 1st order minima from the centre of the screen on either sides of the centre.

Position of minima is expressed by y (which is evaluated from the centre of the screen):

Now, for small θ, it can be said:

sin θ ≈ θ 

⇒ λ  = a sin θ ≈ aθ

⇒ θ  = y/D = λ/a

⇒ y  = λD/a

Width of central maximum is basically twice this value:

⇒ Therefore,   Width of central maximum  = 2λD/a

⇒  And,   Angular width of central maximum  = 2θ = 2λ/a

Types of Diffraction

There are two types of Diffraction:

  • Fresnel Diffraction : The light source and the screen both are at limited good ways from the slit. The episode waves are not equal.
  • Fraunhofer Diffraction : The light source and the screen both are vastly away from the slit with the end goal that the occurrence of light beams are equal.

Types of Diffraction

The constructive interference condition is that the path difference must be equivalent to the integral multiple of the wavelength.

Compton effect can be expressed as the effect observed when x-rays or gamma rays are scattered on a material with increase in wavelength.

The destructive interference condition is that the path difference must be equivalent to the odd integral multiple of half wavelength.

Temporal coherence can be defined as the correlation between field at one point and the field at the same point later some time.

Previous Year Questions

  • For a crystal, the angle of diffraction …  [BITSAT 2008]
  • When light is incident on a diffraction grating the zero order …  [KCET 2004]
  • The width of the diffraction band varies …  [KCET 2006]
  • A parallel monochromatic beam of light is incident normally on a narrow slit …  [JEE Advance 1998]
  • Calculate the wavelength of light used in an interference experiment …  [BITSAT 2015]
  • Two beams of light having intensities …  [JEE Advance 2001]
  • Angular width of the central maxima in the Fraunhofer diffraction …  [NEET 2019]
  • For a parallel beam of monochromatic light of wavelength λ …  [NEET 2015]
  • The angular resolution of a 10 cm diameter telescope at a wavelength of …  [NEET 2005]
  • The ratio of resolving powers of an optical microscope …  [NEET 2017]
  • A beam of light of wavelength 600nm from a distant source falls on a single slit …  [KCET 2004, UPSEE 2006]
  • The aperture diameter of a telescope is 5 m …  [JEE Mains 2020]
  • The angle of minimum deviation for an incident light ray on an equilateral prism …  [KCET 2009]
  • In refraction, light waves are bent on passing from one medium …  [KCET 2021]
  • Two plane wavefronts of light, one incident on a thin convex lens …  [KEAM]

Things to Remember

  • Diffraction is the inclination of a wave produced from a limited source or going through a limited gap to fan out as it spreads. Diffraction results from the impedance of an endless number of waves transmitted by constant circulation of source focus in a few measurements.
  • For monochromatic light,  I(θ) = Io Sin 2 α/α 2  
  • For Bright Fringes,  θ = sin-1 ± (2n+1λ)/2
  • For Dark Fringes,  asinθ = m λ

Sample Questions

Ques: What do you mean by the Compton effect? (1 mark)

Ans: Compton effect, discovered by Arthur Holly Compton, is the increase in the wavelength of the X-rays and the gamma rays which occurs when they are scattered.

Ques: What is fringe width? (2 marks)

Ans: Fringe width is defined as the distance between two successive bright fringes or two successive dark fringes. In the interference pattern, the fringe width is constant for all the fringes. 

Ques: What is meant by phase difference? (2 marks)

Ans: Phase difference is referred to as the difference between any two waves or the particles having the same frequency and starting from the same point. The phase difference is expressed in degrees or radians.

Ques: What is the condition for constructive and destructive interference? (2 marks)

Ans: The condition for constructive interference is that the way contrast ought to be equivalent to an essential variable of the frequency. The condition for destructive interference is that the way contrast ought to be equivalent to an odd fundamental numerous of half frequency.

Ques: Consider a solitary slit diffraction design for a slit width w. It is seen that for the light of frequency 400 nm the point between the primary least and the focal greatest is 4*10 -3 radians. What is the worth of w? (3 marks)

Ans: Dark edges in the diffraction example of the single slit are found at points θ for which w*sinθ = mλ, where m is a number, m = 1, 2, 3, ... . For the principal dim periphery we have w*sinθ = λ. Here we are approached to address this condition for w.

Subtleties of the estimation:

First least: w*sinθ = λ, w = (400 nm)/sin(4*10 -3 radians) = 1*10 -4 m.

Ques: When a monochromatic light source radiates through a 0.2 mm wide slit onto a screen 3.5 m away, the main dim band in the example seems 9.1 mm from the focal point of the brilliant band. What is the frequency of light? (3 marks)

Ans: Dark edges in the diffraction example of a single slit are found at points θ for which w*sinθ = mλ, where m is a whole number, m = 1, 2, 3, ... . For the primary dim periphery we have w*sinθ = λ. Here we are approached to settle this condition for λ.

z = 9.1 mm = 9.1*10 -3 m.

w = 0.2 mm = 2*10 -4 m.

L >> z, thus sinθ ~ z/L and λ = zw/(mL).

λ = (9.1*10 -3 m)(2*10 -4 m)/(3.5 m).

λ = 5.2*10 -7 m = 520 nm.

Ques: What is the distinction between the Fresnel and Fraunhofer class of diffraction? (2 marks)

Ans:  The light source and the screen both are at limited good ways from the slit for Fresnel diffraction while the distances are endless for Fraunhofer diffraction. The occurrence of light beams is equal (plane wavefront) for the last mentioned. For Fresnel diffraction, the occurrence light can have a circular or barrel-shaped wavefront.

Ques: What is stage contrast? (2 marks)

Ans: The stage distinction is characterized as the contrast between any two waves or the particles having a similar recurrence and beginning from a similar point. It is communicated in degrees or radians.

Ques: What is single-slit diffraction? (2 marks)

Ans: If monochromatic light falls on a thin slit having a width practically identical to the frequency of the episode light, a trademark example of dull and splendid districts is acquired on a screen set before the slit. The waves from each place the slit begin to spread in stage however procure a stage contrast on the screen as they cross various distances. The noticed example is brought about by the connection among force and way contrast.

Ques: What are the conditions for diffraction? (2 marks)

Ans: The conditions for diffraction are as follows:

  • The episode light ought to be monochromatic.
  • The slit width ought to be practically identical to the frequency of episode light.

Ques: Fraunhofer diffraction at a solitary slit is performed utilizing a 700 nm light. On the off chance that the primary dull periphery shows up at a point of 300, discover the slit width. (3 marks)

Ans: Using the diffraction equation for a solitary slit of width a, the nth dull periphery happens for,

a sin θ= nλ

At point θ=300, the main dull periphery is found. Utilizing n=1 and λ= 700 nm=700 X 10 -9 m,

a sin 300=1 X 700 X 10 -9 m

a=14 X 10 -7 m

The slit width is 1400 nm.

For Latest Updates on Upcoming Board Exams, Click Here: https://t.me/class_10_12_board_updates

Also Check Out:

 
   
 

CBSE CLASS XII Related Questions

1. a capillary tube of radius r is dipped inside a large vessel of water. the mass of water raised above water level is m. if the radius of capillary is doubled, the mass of water inside capillary will be.

\(\frac M4\)

2. A series LCR circuit with R = 20 W, L = 1.5 H and C = 35 μF is connected to a variable-frequency 200 V ac supply. When the frequency of the supply equals the natural frequency of the circuit, what is the average power transferred to the circuit in one complete cycle?

3. (a) a circular coil of 30 turns and radius 8.0 cm carrying a current of 6.0 a is suspended vertically in a uniform horizontal magnetic field of magnitude 1.0 t. the field lines make an angle of 60° with the normal of the coil. calculate the magnitude of the counter torque that must be applied to prevent the coil from turning.  (b) would your answer change, if the circular coil in (a) were replaced by a planar coil of some irregular shape that encloses the same area (all other particulars are also unaltered.), 4. a series lcr circuit connected to a variable frequency 230 v source. l = 5.0 h, c = 80mf, r = 40 ω. (a) determine the source frequency which drives the circuit in resonance. (b) obtain the impedance of the circuit and the amplitude of current at the resonating frequency. (c) determine the rms potential drops across the three elements of the circuit. show that the potential drop across the lc combination is zero at the resonating frequency, 5. a boy of mass 50 kg is standing at one end of a, boat of length 9 m and mass 400 kg. he runs to the other, end. the distance through which the centre of mass of the boat boy system moves is, 6. a convex lens of glass is immersed in water compared to its power in air, its power in water will.

  • decrease for red light increase for violet light

Similar Physics Concepts

Exam Pattern

Paper Analysis

Physics Syllabus

Chemistry Syllabus

Mathematics Syllabus

Biology Syllabus

English Syllabus

Physical Education Syllabus

Computer Science Syllabus

Economics Syllabus

Business Studies Syllabus

Political Science Syllabus

Physics Practical

Chemistry Practical

History Syllabus

Accountancy Syllabus

Chapter Wise Weightage

Geography Syllabus

Biology Practical

SUBSCRIBE TO OUR NEWS LETTER

downloadapp_banner image

.

( ) ( ) with the phasor sum

 
Diffraction pattern from a single slit

The sketch shows the view from above a single slit. Let's assume that the slit is constant width and very tall compared with that width, so that we can consider the system as two-dimensional. With light at normal incidence, the pattern is symmetrical about the axis of the slit. On a distant screen, the light arriving on the axis from all points in the slit has travelled an equal distance from the slit, so the centre of the pattern is a maximum. The next question is what determines its width.

Notice the broad central maximum, and the equally spaced, successively weaker maxima on either side.

First order minima

This animated sketch shows the angle of the first order minima: the first minimum on either side of the central maximum. We call the slit width , and we imagine it divided into two equal halves. Using the , we consider a point at the very top of the slit, and another point a distance /2 below it, a point at the very top of the lower half of the slit. Consider parallel rays from both points, at angle to the axis of symmetry. (Why parallel? Because the screen is distant. Typically in diffraction experiments, the slit is ~ 10 µm wide, while the distance to the screen might be ~ 1 m.)

The ray from the distance /2 below has to travel an extra distance (  sin  /2). If this distance is half a wavelength, . if  sin    = 

then they are π/2 out of phase and they interfere destructively. Now, for every point in the top half of the slit, there is one in the bottom half a distance /2 below and, at the angle that satisfies  sin    =  , they all interfere destructively.

So the first mimimum has sin    =  / . On the other side of the axis of symmetry, sin    =  / is also a minimum. These two minima limit the broad central maximum.

 

Higher order minima

An argument like the one applies if, in our imagination, we divide the slit into any even number of equal slices. THe diagram shows a division into four. Each half is divided into quarters, and light from a source in the first quarter cancels that from one in the second quarter. Similarly, sources in the third quarter are nullified by those in the fourth quarter. So this diagram represents the second order minima, where sin    =  / , or sin    =  / . For the nth order minima, we have

 sin    =  , where n is an integer, but not zero.

Remember that, on the axis where   =  0, there is a minimum, so the minima are equally spaced in sin  , except either side of the central maximum.

We can note too that, for light diffracting the throught slits, the slit is usually much wider than a wavelength, so the pattern is usually very small, so the approximation that sin  = is usually good.

Next we calculate how the intensity varies with sin  .

 

Intensity I ( θ )

.

The slit of width a is divided into N slits, each of width δa . The path difference between rays from successive slices of the slit are equal, and so too are the angles between successive phasors . The slices of the slit all have equal width and length, so the lengths of all the phasors are equal. For a large number of slices, the phasors approximate the arc of a circle, as shown in red. We'll call the angle that this subtends 2 α . Now 2 α is also the phase difference between the first and the last phasor, which is 2π a  sin  θ / λ .

The phasor sum has magnitude A , which we can write as R .sin  α , where R is the radius of the arc formed by the phasor sum. On the axis, where the phasors are all in phase, the phasor sum is the straight line shown in red at right. This is the amplitude of the diffraction patter at θ /= 0, which we call A 0 . By the definition of angle, 2 α  =  A 0 / R , which gives us R  =  A 0 /2 α . So the magnitude of the phasor sum is

Graphing ( ) with the phasor sum
( ) and

As we remarked when looking at the intenstiy of Young's experiement in Interference , the eye does not respond linearly to intensity. To my eyes, at least, the difference in brightness seems much less than the difference in the intensity graph.

Varying the slit width

DIffraction effects are most noticeable when the slit with a is not very much larger than the wavelength λ . This apparatus allows us to vary the slit width, but the pattern is still projected on a distant screen to make the diffraction effects cl

Young's experiment with finite slit width.

d a
a

In the Young's experiment in Interference , we didn't mention the effect of finite slit width. From the equations above, we can see that, if d is an integral number of times the slit width ( d = na ), then the n th interference fringe is absent: neither slit radiates power at this angle so there are no rays for constructive interference.

a
a d = 3a

Note that the envelope for the interference pattern has four times the magnitude of the diffraction pattern for a single slit. The integral of the combined interference pattern (the area under the red curve) must be twice the integral of diffraction pattern (the area under the black curve), because two slits admit twice as much light, so its maximum must be four times the maximum of the diffraction pattern. We show this experimentally in Interference with single photons. (In the photograph above, it's hard to see by eye that the factor is four. However, note that the interference peaks are white, because of extreme saturation of the camera.)

Further reading.


.

Compare Young’s Double Slit Interference Pattern and Single Slit Diffraction Pattern. - Physics

Advertisements.

Compare Young’s Double Slit Interference Pattern and Single Slit Diffraction Pattern.

Solution Show Solution

 
i. For a common laboratory setup, the slits in Young’s double-slit experiment are much thinner than their separation. They are usually obtained by using a biprism or a Lloyd’s mirror. The separation between the slits is a few mm only. The single slit used to obtain the diffraction pattern is usually of width less than 1 mm.
ii. Size of the pattern obtained: With the best possible setup, the observer can usually see about 30 to 40 equally spaced bright and dark fringes of nearly the same brightness. Size of the pattern obtained: Taken on either side, the observer can see around 20 to 30 fringes with the central fringe being the brightest.
iii. W = `(λ  "D")/"d"` W = `(λ  "D")/"a"` Except for the central bright fringe
iv.
a. Phase difference, Φ between extreme rays: n(2π)  Phase difference, Φ between extreme rays: `("n" + 1/2)` (2π) OR (2n + 1)π
b. Angular position, θ: n`(λ/"d")` Angular position, θ: `("n" + 1/2)(λ/"a")` OR `((2"n" + 1)λ)/(2"a")`
c. Path difference, ∆l between extreme rays: nλ Path difference, ∆l between extreme rays: nλ
d. Distance from the central bright spot, y: n`((λ"D")/"d")` = nW Distance from the central bright spot, y: `("n" + 1/2)((λ"D")/"a") = ("n" + 1/2)`W
v. dark fringe
a. Phase difference, Φ between extreme rays: `("n" - 1/2) (2pi)` OR (2n - 1)π  Phase difference, Φ between extreme rays: n(2π)
b. Angular position, θ: `("n" - 1/2)(λ/"d")` OR (2n - 1)`λ/(2"d")` Angular position, θ: n`(λ/"a")`
c. Path difference, ∆l between extreme rays: `("n" - 1/2)λ` OR `(2"n" - 1)λ/2` Path difference, ∆l between extreme rays: nλ
d. Distance from the central bright spot, y': `("n" - 1/2)((λ "D")/"d") = ("n" - 1/2)`W Distance from the central bright spot, y': n`((λ"D")/"a")` = nW 

RELATED QUESTIONS

In Fraunhoffer diffraction by a narrow slit, a screen is placed at a distance of 2 m from the lens to obtain the diffraction pattern. If the slit width is 0.2 mm and the first minimum is 5 mm on either side of the central maximum, find the wavelength of light. 

What is the diffraction of light? How does it differ from interference? What are Fraunhofer and Fresnel diffractions?

Derive the conditions for bright and dark fringes produced due to diffraction by a single slit.

In a biprism experiment, the fringes are observed in the focal plane of the eyepiece at a distance of 1.2 m from the slits. The distance between the central bright and the 20th bright band is 0.4 cm. When a convex lens is placed between the biprism and the eyepiece, 90 cm from the eyepiece, the distance between the two virtual magnified images is found to be 0.9 cm. Determine the wavelength of light used.

What must be the ratio of the slit width to the wavelength of light for a single slit to have the first diffraction minimum at 45.0°?

A star is emitting light at the wavelength of 5000 Å. Determine the limit of resolution of a telescope having an objective of a diameter of 200 inch.

The bending of a beam of light around corners of obstacle is called ______ 

In a single slit diffraction pattern, the first minima obtained with the red light of wavelength 6600 A.U. coincides with the first maxima f some other wavelength λ then is ______  

What must be the ratio of the slit width to the wavelength for a single slit, to have the first diffraction minimum at 45˚? 

What is the difference between Fresnel and Fraunhofer diffraction? 

Explain experimental setup for Fraunhofer diffraction with neat diagram. 

Explain Fraunhofer diffraction at a single slit with a neat ray diagram. Obtain an expression for the width of the central bright fringe. 

In diffraction experiment, from a single slit, the angular width of the central maxima does not depend upon ____________.

Light of wavelength 'λ' is incident on a single slit of width 'a' and the distance between slit and screen is 'D'. In diffraction pattern, if slit width is equal to the width of the central maximum then 'D' is equal to ______.

The luminous border that surrounds the profile of a mountain just before sun rises behind it, is an example of ______.

A slit of width 'a' illuminated by white light. The first diffraction minimum for light of wavelength 6500 Å is formed at θ = 30°, then 'a' is (sin 30° = 0.5).

In Fresnel's biprism experiment, when the distance between the slit aperture and eye is increased, then distance between the fringes ____________.

The first minimum of a single slit diffraction pattern is observed at angle 2° with a light of wavelength 510 nm. The width of this slit is ____________.

When two coherent sources in Young's experiment are far apart, then interference pattern ______  

A parallel beam of monochromatic light of wavelength 5 × 10 -7 m is incident normally on a single narrow slit of width 10 -3 mm. At what angle of diffraction, the first minima are observed? 

In Young's double slit experiment, the fringe width is 12 mm. If the entire arrangement is placed in water of refractive index `4/3`, then the fringe width becomes (in mm) ______.

The angular separation of the central maximum in the Fraunhofer diffraction pattern is measured. The slit is illuminated by the light of wavelength 6000 Å. If the slit is illuminated by light of another wavelength, the angular separation decreases by 30%. The wavelength of light used is ______.

Two coherent monochromatic light beams of amplitudes E 10 and E 20 produce an interference pattern. The ratio of the intensities of the maxim a and minima in the interference pattern is ______.

In a biprism experiment, the slit is illuminated by red light of wavelength 6400 A and the crosswire of eyepiece is adjusted to the centre of 3 rd bright band. By using blue light it is found that 4 th bright band is at the centre of the cross wire. Calculate the wavelength of blue light.

State the characteristics of a single slit diffraction pattern.

Using the geometry of the double slit experiment, derive the expression for fringe width of interference bands.

Download the Shaalaa app from the Google Play Store

  • Maharashtra Board Question Bank with Solutions (Official)
  • Balbharati Solutions (Maharashtra)
  • Samacheer Kalvi Solutions (Tamil Nadu)
  • NCERT Solutions
  • RD Sharma Solutions
  • RD Sharma Class 10 Solutions
  • RD Sharma Class 9 Solutions
  • Lakhmir Singh Solutions
  • TS Grewal Solutions
  • ICSE Class 10 Solutions
  • Selina ICSE Concise Solutions
  • Frank ICSE Solutions
  • ML Aggarwal Solutions
  • NCERT Solutions for Class 12 Maths
  • NCERT Solutions for Class 12 Physics
  • NCERT Solutions for Class 12 Chemistry
  • NCERT Solutions for Class 12 Biology
  • NCERT Solutions for Class 11 Maths
  • NCERT Solutions for Class 11 Physics
  • NCERT Solutions for Class 11 Chemistry
  • NCERT Solutions for Class 11 Biology
  • NCERT Solutions for Class 10 Maths
  • NCERT Solutions for Class 10 Science
  • NCERT Solutions for Class 9 Maths
  • NCERT Solutions for Class 9 Science
  • CBSE Study Material
  • Maharashtra State Board Study Material
  • Tamil Nadu State Board Study Material
  • CISCE ICSE / ISC Study Material
  • Mumbai University Engineering Study Material
  • CBSE Previous Year Question Paper With Solution for Class 12 Arts
  • CBSE Previous Year Question Paper With Solution for Class 12 Commerce
  • CBSE Previous Year Question Paper With Solution for Class 12 Science
  • CBSE Previous Year Question Paper With Solution for Class 10
  • Maharashtra State Board Previous Year Question Paper With Solution for Class 12 Arts
  • Maharashtra State Board Previous Year Question Paper With Solution for Class 12 Commerce
  • Maharashtra State Board Previous Year Question Paper With Solution for Class 12 Science
  • Maharashtra State Board Previous Year Question Paper With Solution for Class 10
  • CISCE ICSE / ISC Board Previous Year Question Paper With Solution for Class 12 Arts
  • CISCE ICSE / ISC Board Previous Year Question Paper With Solution for Class 12 Commerce
  • CISCE ICSE / ISC Board Previous Year Question Paper With Solution for Class 12 Science
  • CISCE ICSE / ISC Board Previous Year Question Paper With Solution for Class 10
  • Entrance Exams
  • Video Tutorials
  • Question Papers
  • Question Bank Solutions
  • Question Search (beta)
  • More Quick Links
  • Privacy Policy
  • Terms and Conditions
  • Shaalaa App
  • Ad-free Subscriptions

Select a course

  • Class 1 - 4
  • Class 5 - 8
  • Class 9 - 10
  • Class 11 - 12
  • Search by Text or Image
  • Textbook Solutions
  • Study Material
  • Remove All Ads
  • Change mode

PhET Home Page

  • Sign in / Register
  • Administration
  • Edit profile

young's single slit experiment class 12

The PhET website does not support your browser. We recommend using the latest version of Chrome, Firefox, Safari, or Edge.

  • Interference: Young’s Double Slit Experiment
  • Active page

Optics of Class 12

Interference: young’s double slit experiment .

It was carried out in 1802 by the English scientist Thomas Young to prove the wave nature of light. 

Two slits S 1 and S 2 are made in an opaque screen, parallel and very close to each other. These two are illuminated by another narrow slits S which in turn lit by a bright source. Light wave spread out from S and fall on both S 1 and S 2 which behave like coherent sources. Note that the coherent sources are derived from the same source. In this way, any phase change in S occurs in both S 1 and S 2 . The phase difference (φ 1 - φ 2 ) between S 1 and S 2 is unaffected and remain constant. 

Light now spreads out from both S 1 and S 2 falls on a screen. It is essential that the waves from the two sources overlap on the same part of the screen. If one slit is covered up, the other produces a wide smoothly illuminated patch on the screen. But when both slits are open, the patch is seen to be crossed by dark and bright bands called interference fringes.

Condition for bright fringes or maxima 

φ = 2nπ  or   path difference ,   p  = nλ   where n = 0, 1, 2, ……

Interference: Young’s Double Slit Experiment

Condition for dark fringes or minima 

φ = (2n – 1)πorpath difference,  p = (n - 1/2)λ

where n = 1, 2, 3, …………

Interference: Young’s Double Slit Experiment

The relation between phase difference (φ) and path difference (p) is given by 

φ = 2π/λ p (15.32)

How to find the position of the nth maxima or minima on the screen?

Interference: Young’s Double Slit Experiment

Let P be the position of the nth maxima on the screen. The  two waves arriving at P follow the path S 1 P and S 2 P, thus the path difference between the two waves is 

p = S 1 P – S 1 P = d sin θ 

From experimental conditions, we know that D >> d, therefore, the angle θ is small, 

thus sinθ ≈ tanθ = Yn/D

∴p = d sinθ = d tan θ  = d (Yn/D) ⇒   y n = pD/d

For nth maxima 

p = nλ 

∴y n =  nλ D/d where  n = 0, 1, 2, ……..(15.33)

For nth minima 

p = (n - 1/2)λ

∴y n  = (n - 1/2)λD/d where n = 1, 2, 3, ……(15.34)

 Note that the nth minima comes before the nth maxima.

Fringe Width 

It is defined as the distance between two successive maxima or minima.

Interference: Young’s Double Slit Experiment

Example 15.23

In a YDSE , if D = 2 m; d = 6 mm and λ = 6000 Å, then 

(a)find the fringe width 

(b)find the position of the 3 rd maxima 

(c)find the position of the 2 nd minima 

Solution 

(a)Using equation (15.33),

ω = λD/d   = (6000 x 10 -10 )(2)/6 x 10 -3 = 0.2 mm 

(b)Position of 3 rd maxima is obtained by putting n = 3 in the equation (15.33),

y 3 = 3 λD = 3ω = 3(0.2) = 0.6 mm 

(c)Position of 2 nd minima is obtained by putting n = 2 in the equation (15.34),

Interference: Young’s Double Slit Experiment

Example 15.24

What is the effect on the interference fringes in a YDSE due to each of the following operations.

(a)the screen is moved away from the plane of the slits 

(b)the (monochromatic) source is replaced by another (monochromatic) source of shorter wavelength 

(c)the separation between the two slits is increased 

(d)the monochromatic source is replaced by source of white light 

(e)the whole experiment is carried out in a medium of refractive index μ 

(a)Angular separation (= λ/d ) of the fringes remains constant. But the linear separation or fringe width increases in proportion to the distance (D) from the screen. 

(b)As λ decreases, fringe width (ω ∝ λ) decreases 

(c)As d increases, fringe width (ω ∝ 1/d) decreases 

(d)The interference pattern due to different component colours of white light overlap (in-coherently). The central bright fringes of different colours are at the same position. Therefore, the central fringe is white. Since blue colour has the lower λ, the fringe closest on either side of the central white fringe is blue; the farthest is red. 

Interference: Young’s Double Slit Experiment

Thus, fringe width decreases by μ. 

Example  15.25

Light from a source consists of two wavelength λ 1 = 6500Å and λ 2 = 5200Å. If D = 2m and d = 6.5 mm find the minimum value of y (≠ 0) where the maxima of both the wavelengths coincide. 

Let n 1 th maxima of λ 1 coincides with n 2 th maxima of λ 2 . 

Then yn1 = yn2

Interference: Young’s Double Slit Experiment

Thus, fourth maxima of λ 1 coincides with fifth maxima of λ 2 . 

The minimum value of y (≠ 0) is given by 

Interference: Young’s Double Slit Experiment

Optical Path

It is defined as distance travelled by light in vacuum in the same time in which it travels a given path length in a medium. If light travels a path length d in a medium at speed c, the time taken by it will be (d/c). So optical path length−

L = c o × [d/c] = μd(because  μ = c0/c )(15.36)

Since for all media μ > 1, optical path length is always greater than geometrical path length. When two light waves arrive at a point by travelling different distances in different media, the phase difference between the two is related by their optical path difference instead of simply path difference. 

Phase Difference = 2π/ λ (optical path difference) = 2π/ λ L

Fringe Shift 

When a transparent film of thickness t and refractive index μ is introduced in front of one of the slits, the fringe pattern shifts in the direction where the film is placed. 

How much is the fringe shift?

Consider  the YDSE arrangement shown in the figure.

The optical path difference is given by 

p = [(S2P – t) + μt] – S1

orp = (S2P – S1P) + t(μ - 1) 

SinceS2P – S1P = d sin θ 

∴p = d sin θ + t (μ - 1)

As sin θ ≈ tan θ =

∴p =

Interference: Young’s Double Slit Experiment

In the absence of film, the position of the nth maxima is given by equation (15.34)

y n = n λD/d

Therefore, the fringe shift is given by 

FS = y n - y′ n = (μ - 1) tD/d (15.37)

Note that the shift is in the direction where the film is introduced.

Example 15.26

Ιn a YDSE, λ = 6000Å, D = 2 m, d = 6 mm. When a film of refractive index 1.5 is introduced in front of the lower slit, the third maxima shifts to the origin. 

(a)Find the thickness of the film 

(b)Find the positions of the fourth maxima 

(a)Since 3 rd minima shifts to the origin, therefore, the fringe shift is given by 

FS = y 3 = 3 λD/d

From equation (15.37), we know 

FS = (μ - 1) tD/d

∴(μ - 1) tD/d = 3 λD/d

or t = 3 λ/μ - 1

Here λ = 0.6 × 10 -6 m;  μ = 1.5 

∴t = 3(0.6 x 10 -6 )/1.5 -1 = 3.6 μm

(b)There are two positions of 4th maxima: one above and the other below the origin. 

y4 = 1ω = λD/d = 0.2 mm

y′4 = -7ω = -7λD/d = 1.4 mm

Intensity Distribution 

When two coherent light waves of intensity I 1 and I 2 with a constant phase difference φ superimpose, then the resultant intensity is given by 

I = I 1 + I 2 + 2 √I 1 I 2 cosφ

In YDSE, usually the intensities I 1 and I 2 are equal 

i.e.I 1 = I 2 = I o

Then I = 2I o ( 1+ cos φ) 

orI = 4I o cos 2 (φ/2)(15.38) 

For maxima:φ = 2nπ ⇒I max = 4I o

For minima:φ = (2n – 1)π⇒I min = 0 

Interference: Young’s Double Slit Experiment

Example 15.27

The intensity of the light coming from one of the slits in YDSE is double the intensity from the other slit. Find the ratio of the maximum intensity to the minimum intensity in the interference fringe pattern observed. 

Interference: Young’s Double Slit Experiment

SinceI 1 = 2I o and I 2 = I o ,  therefore 

Interference: Young’s Double Slit Experiment

Example 15.28

In a YDSE, λ = 60 nm; D = 2m; λ = 6 mm. Find the positions of a point lying between third maxima and third minima where the intensity is three-fourth of the maximum intensity on the screen. 

Using equation (15.32) 

I = 4I o cos 2 φ/2

here I = 3/4 (4I o ) = 3I o

∴cos φ/2 = √3/2

Thus,φ/2 = nπ ± π/6

or φ = 2nπ ± π/3

Interference: Young’s Double Slit Experiment

For the point lying between third minima and third maxima, 

Interference: Young’s Double Slit Experiment

or y 3 = 17/6 λD/d

Putting λ = 0.6 × 10 -6 m;  D = 2m;  d = 6 mm 

Interference: Young’s Double Slit Experiment

  • Geometrical Optics
  • Reflection from a Plane-surface
  • Reflection from Curved Surfaces
  • Refraction Through Plane Surfaces
  • Refraction on Curved Surfaces
  • Wave Optics

center banner

Talk to Our counsellor

Talk to Our counsellor

  • He placed a screen that had two slits cut into it in front of a monochromatic ( single color ) light.
  • The results of Young's Double Slit Experiment should be very different if light is a wave or a particle.
  • Let’s look at what the results would be in both situations, and then see how this experiment supports the wave model.

If light is a particle…

We set up our screen and shine a bunch of monochromatic light onto it.

  • Imagine it as being almost as though we are spraying paint from a spray can through the openings.
  • Since they are little particles they will make a pattern of two exact lines on the viewing screen ( Figure 1 ).

If light is a wave…

If light is a wave, everything starts the same way, but results we get are very different.

  • Remember, diffraction is when light passes through a small opening and starts to spread out. This will happen from both openings ( Figure 2 ).
  • Where crest meets crest , there will be constructive interference and the waves will make it to the viewing screen as a bright spot .
  • Where crest meets trough there will be destructive interference that cancel each other out… a black spot will appear on the screen.
  • When this experiment is performed we actually see this, as shown in Figure 3 .

We must conclude that light is made up of waves, since particles can not diffract.

Calculations

When you set up this sort of an apparatus, there is actually a way for you to calculate where the bright lines (called fringes ) will appear.

  • There is always a middle line, which is the brightest. We call it the central fringe .
  • The central fringe is n = 0 .
  • The fringe to either side of the central fringe has an order of n = 1 (the first order fringe ).
  • The order of the next fringe out on either side is n = 2 (the second order fringe ).
  • And so on, as shown in Figure 4 .

The formula that we will use to figure out problems involving double slit experiments is easy to mix up, so make sure you study it carefully.

λ = wavelength of light used (m) x = distance from central fringe (m) d = distance between the slits (m) n = the order of the fringe L = length from the screen with slits to the viewing screen (m)

It is very easy to mix up the measurements of x, d, and L.

  • Make sure to look at Figure 5 and see the different things each is measuring.
  • If you mix up x and d it's not so bad, since they are both on top in the formula. If you were to mix them up with L, you would get the wrong answer.
  • Almost all questions that you will see for this formula just involve sorting out what each variable is... you might find it helpful to write out a list of givens.

Example 1 : A pair of screens are placed 13.7m apart. A third order fringe is seen on the screen 2.50cm from the central fringe. If the slits were cut 0.0960 cm apart, determine the wavelength of this light. Roughly what colour is it?

Just to make sure you’ve got all the numbers from the question matched with the correct variables… L = 13.7 m n = 3 x = 2.50cm = 0.0250 m d = 0.0960cm = 9.60e-4 m It’s probably a yellow light being used given the wavelength we've measured.

If a white light is used in the double slit experiment, the different colours will be split up on the viewing screen according to their wavelengths.

  • The violet end of the spectrum (with the shortest wavelengths) is closer to the central fringe, with the other colours being further away in order.

There is also a version of the formula where you measure the angle between the central fringe and whatever fringe you are measuring.

  • The formula works the same way, with the only difference being that we measure the angle instead of x and L.
  • Make sure that your calculator is in degree mode before using this version of the formula.

Example 2 : If a yellow light with a wavelength of 540 nm shines on a double slit with the slits cut 0.0100 mm apart, determine what angle you should look away from the central fringe to see the second order fringe?

Do not forget to: Change the wavelength into metres. Change the slit separation into metres. "Second order" is a perfect number and has an infinite number of sig digs.

The Single Slit

A surprising experiment is that you can get the same effect from using a single slit instead of a double slit.

  • In Figure 7 , the blue path has to travel further than the red path... if this difference is equal to half a wavelength, they will meet each other out of sync .
  • If they meet crest to crest or trough to trough they will be in sync , but if they meet crest to trough they will be out of sync .
  • Being in sync will result in constructive interference , while meeting out of sync will result in destructive interference .
  • After the first couple of fringes (n = 1 and 2), the edges start getting really fuzzy, so you have a hard time measuring anything.
  • The only real difference in calculations is that "d" is now the width of the single opening.
  • If two slits work better than one, would more than two slits work better? This is a question that we will answer in the next section.

Example 3 : For a single slit experiment apparatus like the one described above, determine how far from the central fringe the first order violet (λ = 350nm) and red (λ = 700nm) colours will appear if the screen is 10 m away and the slit is 0.050 cm wide.

We need to solve the formula for “x”, the distance from the central fringe. For the violet light… For the red light…

Logo for British Columbia/Yukon Open Authoring Platform

Want to create or adapt books like this? Learn more about how Pressbooks supports open publishing practices.

Chapter 12 – Wave Optics

12.3 Young’s Double Slit Experiment

  • Explain the phenomena of interference.
  • Define constructive interference for a double slit and destructive interference for a double slit.

Although Christiaan Huygens thought that light was a wave, Isaac Newton did not. Newton felt that there were other explanations for colour, and for the interference and diffraction effects that were observable at the time. Owing to Newton’s tremendous stature, his view generally prevailed. The fact that Huygens’s principle worked was not considered evidence that was direct enough to prove that light is a wave. The acceptance of the wave character of light came many years later when, in 1801, the English physicist and physician Thomas Young (1773–1829) did his now-classic double slit experiment (see Figure 1 ).

A beam of light strikes a wall through which a pair of vertical slits is cut. On the other side of the wall, another wall shows a pattern of equally spaced vertical lines of light that are of the same height as the slit.

Why do we not ordinarily observe wave behaviour for light, such as observed in Young’s double slit experiment? First, light must interact with something small, such as the closely spaced slits used by Young, to show pronounced wave effects. Furthermore, Young first passed light from a single source (the Sun) through a single slit to make the light somewhat coherent. By coherent , we mean waves are in phase or have a definite phase relationship. Incoherent means the waves have random phase relationships. Why did Young then pass the light through a double slit? The answer to this question is that two slits provide two coherent light sources that then interfere constructively or destructively. Young used sunlight, where each wavelength forms its own pattern, making the effect more difficult to see. We illustrate the double slit experiment with monochromatic (single colour, single wavelength,   λ ) light to clarify the effect. Figure 2 shows the pure constructive and destructive interference of two waves having the same wavelength and amplitude.

Figure a shows three sine waves with the same wavelength arranged one above the other. The peaks and troughs of each wave are aligned with those of the other waves. The top two waves are labeled wave one and wave two and the bottom wave is labeled resultant. The amplitude of waves one and two are labeled x and the amplitude of the resultant wave is labeled two x. Figure b shows a similar situation, except that the peaks of wave two now align with the troughs of wave one. The resultant wave is now a straight horizontal line on the x axis; that is, the line y equals zero.

When light passes through narrow slits, it is diffracted into semicircular waves, as shown in Figure 3 (a). Pure constructive interference occurs where the waves are crest to crest or trough to trough. Pure destructive interference occurs where they are crest to trough. The light must fall on a screen and be scattered into our eyes for us to see the pattern. An analogous pattern for water waves is shown in Figure 3 (b). Note that regions of constructive and destructive interference move out from the slits at well-defined angles to the original beam. These angles depend on wavelength and the distance between the slits, as we shall see below.

The figure contains three parts. The first part is a drawing that shows parallel wavefronts approaching a wall from the left. Crests are shown as continuous lines, and troughs are shown as dotted lines. Two light rays pass through small slits in the wall and emerge in a fan-like pattern from two slits. These lines fan out to the right until they hit the right-hand wall. The points where these fan lines hit the right-hand wall are alternately labeled min and max. The min points correspond to lines that connect the overlapping crests and troughs, and the max points correspond to the lines that connect the overlapping crests. The second drawing is a view from above of a pool of water with semicircular wavefronts emanating from two points on the left side of the pool that are arranged one above the other. These semicircular waves overlap with each other and form a pattern much like the pattern formed by the arcs in the first image. The third drawing shows a vertical dotted line, with some dots appearing brighter than other dots. The brightness pattern is symmetric about the midpoint of this line. The dots near the midpoint are the brightest. As you move from the midpoint up, or down, the dots become progressively dimmer until there seems to be a dot missing. If you progress still farther from the midpoint, the dots appear again and get brighter, but are much less bright than the central dots. If you progress still farther from the midpoint, the dots get dimmer again and then disappear again, which is where the dotted line stops.

To understand the double slit interference pattern, we consider how two waves travel from the slits to the screen, as illustrated in Figure 4 . Each slit is a different distance from a given point on the screen. Thus different numbers of wavelengths fit into each path. Waves start out from the slits in phase (crest to crest), but they may end up out of phase (crest to trough) at the screen if the paths differ in length by half a wavelength, interfering destructively as shown in Figure 4 (a). If the paths differ by a whole wavelength, then the waves arrive in phase (crest to crest) at the screen, interfering constructively as shown in Figure 4 (b). More generally, if the paths taken by the two waves differ by any half-integral number of wavelengths ( 1/2λ, 3/2 λ, 5/2 λ …   etc.) then destructive interference occurs. Similarly, if the paths taken by the two waves differ by any integral number of wavelengths (1 λ, 2λ, 3λ…  , etc.), then constructive interference occurs.

Take-Home Experiment: Using Fingers as Slits

Look at a light, such as a street lamp or incandescent bulb, through the narrow gap between two fingers held close together. What type of pattern do you see? How does it change when you allow the fingers to move a little farther apart? Is it more distinct for a monochromatic source, such as the yellow light from a sodium vapour lamp, than for an incandescent bulb?

Both parts of the figure show a schematic of a double slit experiment. Two waves, each of which is emitted from a different slit, propagate from the slits to the screen. In the first schematic, when the waves meet on the screen, one of the waves is at a maximum whereas the other is at a minimum. This schematic is labeled dark (destructive interference). In the second schematic, when the waves meet on the screen, both waves are at a minimum.. This schematic is labeled bright (constructive interference).

Figure 5 below shows how to determine the path length difference for waves traveling from two slits to a common point on a screen. If the screen is a large distance away compared with the distance between the slits, then the angle  θ between the path and a line from the slits to the screen (see the figure) is nearly the same for each path. The difference between the paths is shown in the figure; simple trigonometry shows it to be dsinθ, where  d is the distance between the slits. To obtain constructive interference for a double slit , the path length difference must be an integral multiple of the wavelength, or

Similarly, to obtain destructive interference for a double slit , the path length difference must be a half-integral multiple of the wavelength, or

where  λ  is the wavelength of the light,  d is the distance between slits, and θ is the angle from the original direction of the beam as discussed above. We call  m the order of the interference. For example,  m=4 is fourth-order interference.

The figure is a schematic of a double slit experiment, with the scale of the slits enlarged to show the detail. The two slits are on the left, and the screen is on the right. The slits are represented by a thick vertical line with two gaps cut through it a distance d apart. Two rays, one from each slit, angle up and to the right at an angle theta above the horizontal. At the screen, these rays are shown to converge at a common point. The ray from the upper slit is labeled l sub one, and the ray from the lower slit is labeled l sub two. At the slits, a right triangle is drawn, with the thick line between the slits forming the hypotenuse. The hypotenuse is labeled d, which is the distance between the slits. A short piece of the ray from the lower slit is labeled delta l and forms the short side of the right triangle. The long side of the right triangle is formed by a line segment that goes downward and to the right from the upper slit to the lower ray. This line segment is perpendicular to the lower ray, and the angle it makes with the hypotenuse is labeled theta. Beneath this triangle is the formula delta l equals d sine theta.

The equations for double slit interference imply that a series of bright and dark lines are formed. For vertical slits, the light spreads out horizontally on either side of the incident beam into a pattern called interference fringes, illustrated in Figure 6 . The intensity of the bright fringes falls off on either side, being brightest at the centre. The closer the slits are, the more is the spreading of the bright fringes. We can see this by examining the equation

For fixed    λ   and m , the smaller  d is, the larger  θ must be, since dsinθ= mλ.

This is consistent with our contention that wave effects are most noticeable when the object the wave encounters (here, slits a distance  d apart) is small. Small  d gives large θ , hence a large effect.

The figure consists of two parts arranged side-by-side. The diagram on the left side shows a double slit arrangement along with a graph of the resultant intensity pattern on a distant screen. The graph is oriented vertically, so that the intensity peaks grow out and to the left from the screen. The maximum intensity peak is at the center of the screen, and some less intense peaks appear on both sides of the center. These peaks become progressively dimmer upon moving away from the center, and are symmetric with respect to the central peak. The distance from the central maximum to the first dimmer peak is labeled y sub one, and the distance from the central maximum to the second dimmer peak is labeled y sub two. The illustration on the right side shows thick bright horizontal bars on a dark background. Each horizontal bar is aligned with one of the intensity peaks from the first figure.

Example 1: Finding a Wavelength from an Interference Pattern

Suppose you pass light from a He-Ne laser through two slits separated by 0.0100 mm and find that the third bright line on a screen is formed at an angle of 10.95 o relative to the incident beam. What is the wavelength of the light?

The third bright line is due to third-order constructive interference, which means that m=3 . We are given d = 0.0100 mm and θ = 10.95 o . The wavelength can thus be found using the equation d sinθ = m λ for constructive interference.

The equation is  d sinθ = m λ.  Solving for the wavelength  λ  gives

Substituting known values yields

λ = (d sinθ) / m  = (0.0100 x 10 -3 m) (sin 10.95 o )  /  3 = 6.33 x 10 -7 m  or 

λ  =  633 nm

To three digits, this is the wavelength of light emitted by the common He-Ne laser. Not by coincidence, this red colour is similar to that emitted by neon lights. More important, however, is the fact that interference patterns can be used to measure wavelength. Young did this for visible wavelengths. This analytical technique is still widely used to measure electromagnetic spectra. For a given order, the angle for constructive interference increases with λ , so that spectra (measurements of intensity versus wavelength) can be obtained.

Example 2: Calculating Highest Order Possible

Interference patterns do not have an infinite number of lines, since there is a limit to how big  m can be. What is the highest-order constructive interference possible with the system described in the preceding example?

Strategy and Concept

The equation  d sinθ = mλ  for  m = 0,  1, -1, 2, -2, 3, -3, 4, -4, …  describes constructive interference. For fixed values of  d and λ , the larger  m is, the larger  sinθ is. However, the maximum value that sinθ can have is 1, for an angle of 90 o . (Larger angles imply that light goes backward and does not reach the screen at all.) Let us find which  m corresponds to this maximum diffraction angle.

Solving the equation  d sinθ = mλ   for  m gives

Taking sinθ =sin90 o = 1 and substituting the values of  d and  λ, and remembering the values for the metric prefixes, from the preceding example gives

Therefore, the largest integer m can be is 15, or

The number of fringes depends on the wavelength and slit separation. The number of fringes will be very large for large slit separations. However, if the slit separation becomes much greater than the wavelength, the intensity of the interference pattern changes so that the screen has two bright lines cast by the slits, as expected when light behaves like a ray. We also note that the fringes get fainter further away from the centre. Consequently, not all 15 fringes may be observable.

Section Summary

  • Young’s double slit experiment gave definitive proof of the wave character of light.
  • An interference pattern is obtained by the superposition of light from two slits.
  • There is constructive interference when  d sinθ = mλ  for  m = 0, 1, -1, 2, -2, 3, -3, 4, -4, ….(constructive) , where  d is the distance between the slits,  θ is the angle relative to the incident direction, and  m is the order of the interference.
  • There is destructive interference when  d sinθ =  ( m + 1 / 2 )λ   for  m = 0, 1, -1, 2, -2, 3, -3, 4, -4, ….(constructive)

Conceptual Questions

1: Young’s double slit experiment breaks a single light beam into two sources. Would the same pattern be obtained for two independent sources of light, such as the headlights of a distant car? Explain.

2: Suppose you use the same double slit to perform Young’s double slit experiment in air and then repeat the experiment in water. Do the angles to the same parts of the interference pattern get larger or smaller? Does the colour of the light change? Explain.

3: Is it possible to create a situation in which there is only destructive interference? Explain.

4:   Figure 7 shows the central part of the interference pattern for a pure wavelength of red light projected onto a double slit. The pattern is actually a combination of single slit and double slit interference. Note that the bright spots are evenly spaced. Is this a double slit or single slit characteristic? Note that some of the bright spots are dim on either side of the centre. Is this a single slit or double slit characteristic? Which is smaller, the slit width or the separation between slits? Explain your responses.

The figure shows a photo of a horizontal line of equally spaced red dots of light on a black background. The central dot is the brightest and the dots on either side of center are dimmer. The dot intensity decreases to almost zero after moving six dots to the left or right of center. If you continue to move away from the center, the dot brightness increases slightly, although it does not reach the brightness of the central dot. After moving another six dots, or twelve dots in all, to the left or right of center, there is another nearly invisible dot. If you move even farther from the center, the dot intensity again increases, but it does not reach the level of the previous local maximum. At eighteen dots from the center, there is another nearly invisible dot.

Problems & Exercises

  • At what angle is the first-order maximum for 450-nm wavelength blue light falling on double slits separated by 0.0500 mm ?
  • Calculate the angle for the third-order maximum of 580-nm wavelength yellow light falling on double slits separated by 0.100 mm.
  • What is the separation between two slits for which 610-nm orange light has its first maximum at an angle of 30.0  degrees?
  • Find the distance between two slits that produces the first minimum for 410-nm violet light at an angle of 45.0  degrees.
  •  Calculate the wavelength of light that has its third minimum at an angle of 30.0 o when falling on double slits separated by 3.00 μm. Explicitly, show how you follow the steps in  Problem-Solving Strategies for Wave Optics .
  • What is the wavelength of light falling on double slits separated by 2.00 μm if the third-order maximum is at an angle of 60.0 o ?
  • At what angle is the fourth-order maximum for the situation in Problems & Exercises 1 ?
  • What is the highest-order maximum for 400-nm light falling on double slits separated by 25.0 μm ?
  • Find the largest wavelength of light falling on double slits separated by 1.20 μm for which there is a first-order maximum. Is this in the visible part of the spectrum?
  • What is the smallest separation between two slits that will produce a second-order maximum for 720-nm red light?
  • (a) What is the smallest separation between two slits that will produce a second-order maximum for any visible light? (b) For all visible light.
  • (a) If the first-order maximum for pure-wavelength light falling on a double slit is at an angle of 10.0o, at what angle is the second-order maximum? (b) What is the angle of the first minimum? (c) What is the highest-order maximum possible here?
  • Figure 8 shows a double slit located a distance x from a screen, with the distance from the centre of the screen given by y.  When the distance d between the slits is relatively large, there will be numerous bright spots, called fringes. Show that, for small angles (where sinθ≈tanθ, with θ in radians), the distance between fringes is given by Δy = x λ /d.

The figure shows a schematic of a double slit experiment. A double slit is at the left and a screen is at the right. The slits are separated by a distance d. From the midpoint between the slits, a horizontal line labeled x extends to the screen. From the same point, a line angled upward at an angle theta above the horizontal also extends to the screen. The distance between where the horizontal line hits the screen and where the angled line hits the screen is marked y, and the distance between adjacent fringes is given by delta y, which equals x times lambda over d.

14: Using the result of the problem above, calculate the distance between fringes for 633-nm light falling on double slits separated by 0.0800 mm, located 3.00 m from a screen as in Figure 8 .

15: Using the result of the problem two problems prior, find the wavelength of light that produces fringes 7.50 mm apart on a screen 2.00 m from double slits separated by 0.120 mm (see Figure 8 ).

1:  0.516 o

3:  1.22 x10 -6 m

4: 0.290 μm

9: 1200 nm (not visible)

10 :  1.44 μm

11: (a) 760 nm (b) 1520 nm

13: For small angles sinθ ≈ tanθ≈θ when the angle is in radians

For two adjacent fringes we have d sinθ m = m λ

and d sinθ m +1 =  (m+1)  λ

Subtracting these equations gives

 \begin{array}{r @{{}={}}l} \boldsymbol{d (\textbf{sin} \; \theta _{\textbf{m} + 1} - \textbf{sin} \; \theta _{\textbf{m}})} & \boldsymbol{[(m + 1) - m] \lambda} \\[1em] \boldsymbol{d(\theta _{{\textbf{m}} + 1} - \theta _{\textbf{m}})} & \boldsymbol{\lambda} \end{array}

14: 2.37 cm

Douglas College Physics 1207 Copyright © August 22, 2016 by OpenStax is licensed under a Creative Commons Attribution 4.0 International License , except where otherwise noted.

  • Derivation of Physics Formula
  • Youngs Double Slit Experiment Derivation

Young’s Double Slits Experiment Derivation

Introduction to young’s double slits experiment.

During the year 1801, Thomas Young carried out an experiment where the wave and particle nature of light and matter were demonstrated. The schematic diagram of the experimental setup is shown below-

young double slits experiment derivation

A beam of monochromatic light is made incident on the first screen, which contains the slit S 0 . The emerging light then incident on the second screen which consists of two slits namely, S 1 , S 2 . These two slits serve as a source of coherent light. The emerging light waves from these slits interfere to produce an interference pattern on the screen. The interference pattern consists of consecutive bright and dark fringes. The dark fringes are the result of destructive interference and bright fringes are the result of constructive interference.

You may also want to check out these topics given below!

  • Superposition of waves
  • Diffraction
  • Polarisation by scattering

young double slits experiment derivation-1

  • The light falls on the screen at the point P. which is at a distance y from the centre O.
  • The distance between the double-slit system and the screen is L
  • The two slits are separated by the distance d
  • Distance travelled by the light ray from slit 1 to point P on the screen is r 1
  • Distance travelled by the light ray from slit 2 to point P on the screen is r 2
  • Thus, the light ray from slit 2 travels an extra distance of ẟ = r 2 -r 1 than light ray from slit 1.
  • This extra distance is termed as Path difference.

Refer to Figure(3) Applying laws of cosines; we can write –

∵ Cos(90° – θ) = Sinθ

∵ Cos(90° + θ) = -Sinθ

Subtracting equation (2) from (1) we get-

Let us impose the limit that the distance between the double slit system and the screen is much greater than the distance between the slits. That is L >> d. The sum of r 1 and r 2 can be approximated to r 1 + r 2 ≅2r. Thus, the path difference becomes –

In this limit, the two rays r 1 and r 2 are essentially treated as parallel. (See Figure(4))

young double slits experiment derivation-3

Figure(4): Assuming L >> d, The path difference between two rays.

To compare the phase of two waves, the value of path difference (ẟ) plays a crucial role.

Constructive interference is seen when path difference (𝛿) is zero or integral multiple of the wavelength (λ).

(Constructive interference) 𝛿 = dSin𝜃 = mλ —-(5), m = 0, ±⁤1, ±2, ±3, ±4, ±5, ……

Where m is order number. The Zeroth order maximum (m=0)corresponds to the central bright fringe, here 𝜃=0. The first order maxima(m=±⁤1)(bright fringe) are on either side the central fringe.

Similarly, when 𝛿 is an odd integral multiple of λ/2, the resultant fringes will be 180 0 out of phase, thus, forming a dark fringe.

(Destructive interference) 𝛿 = dSin𝜃 = λ—(6), m = 0, ±⁤1, ±2, ±3, ±4, ±5, ……

young double slits experiment derivation

Assuming the distance between the slits are much greater than the wavelength of the incident light, we get-

Substituting it in the constructive and destructive interference condition we can get the position of bright and dark fringes, respectively. The equation is as follows-

Frequently Asked Questions – FAQs

What is meant by interference, name a coherent source of light., which type of fringes are produced from the destructive interference, in which type of interference, the waves will have the same displacement and phase., does the young’s double slit experiment prove the interference of light.

Quiz Image

Put your understanding of this concept to test by answering a few MCQs. Click ‘Start Quiz’ to begin!

Select the correct answer and click on the “Finish” button Check your score and answers at the end of the quiz

Visit BYJU’S for all Physics related queries and study materials

Your result is as below

Request OTP on Voice Call

PHYSICS Related Links

Leave a Comment Cancel reply

Your Mobile number and Email id will not be published. Required fields are marked *

Post My Comment

young's single slit experiment class 12

loads of thanks for providing these free study materials will help students a lot

young's single slit experiment class 12

Register with BYJU'S & Download Free PDFs

Register with byju's & watch live videos.

Logo for University of Central Florida Pressbooks

Chapter 27 Wave Optics

27.3 Young’s Double Slit Experiment

  • Explain the phenomena of interference.
  • Define constructive interference for a double slit and destructive interference for a double slit.

Although Christiaan Huygens thought that light was a wave, Isaac Newton did not. Newton felt that there were other explanations for color, and for the interference and diffraction effects that were observable at the time. Owing to Newton’s tremendous stature, his view generally prevailed. The fact that Huygens’s principle worked was not considered evidence that was direct enough to prove that light is a wave. The acceptance of the wave character of light came many years later when, in 1801, the English physicist and physician Thomas Young (1773–1829) did his now-classic double slit experiment (see Figure 1 ).

A beam of light strikes a wall through which a pair of vertical slits is cut. On the other side of the wall, another wall shows a pattern of equally spaced vertical lines of light that are of the same height as the slit.

When light passes through narrow slits, it is diffracted into semicircular waves, as shown in Figure 3 (a). Pure constructive interference occurs where the waves are crest to crest or trough to trough. Pure destructive interference occurs where they are crest to trough. The light must fall on a screen and be scattered into our eyes for us to see the pattern. An analogous pattern for water waves is shown in Figure 3 (b). Note that regions of constructive and destructive interference move out from the slits at well-defined angles to the original beam. These angles depend on wavelength and the distance between the slits, as we shall see below.

The figure contains three parts. The first part is a drawing that shows parallel wavefronts approaching a wall from the left. Crests are shown as continuous lines, and troughs are shown as dotted lines. Two light rays pass through small slits in the wall and emerge in a fan-like pattern from two slits. These lines fan out to the right until they hit the right-hand wall. The points where these fan lines hit the right-hand wall are alternately labeled min and max. The min points correspond to lines that connect the overlapping crests and troughs, and the max points correspond to the lines that connect the overlapping crests. The second drawing is a view from above of a pool of water with semicircular wavefronts emanating from two points on the left side of the pool that are arranged one above the other. These semicircular waves overlap with each other and form a pattern much like the pattern formed by the arcs in the first image. The third drawing shows a vertical dotted line, with some dots appearing brighter than other dots. The brightness pattern is symmetric about the midpoint of this line. The dots near the midpoint are the brightest. As you move from the midpoint up, or down, the dots become progressively dimmer until there seems to be a dot missing. If you progress still farther from the midpoint, the dots appear again and get brighter, but are much less bright than the central dots. If you progress still farther from the midpoint, the dots get dimmer again and then disappear again, which is where the dotted line stops.

Take-Home Experiment: Using Fingers as Slits

Look at a light, such as a street lamp or incandescent bulb, through the narrow gap between two fingers held close together. What type of pattern do you see? How does it change when you allow the fingers to move a little farther apart? Is it more distinct for a monochromatic source, such as the yellow light from a sodium vapor lamp, than for an incandescent bulb?

Both parts of the figure show a schematic of a double slit experiment. Two waves, each of which is emitted from a different slit, propagate from the slits to the screen. In the first schematic, when the waves meet on the screen, one of the waves is at a maximum whereas the other is at a minimum. This schematic is labeled dark (destructive interference). In the second schematic, when the waves meet on the screen, both waves are at a minimum.. This schematic is labeled bright (constructive interference).

Similarly, to obtain destructive interference for a double slit , the path length difference must be a half-integral multiple of the wavelength, or

\boldsymbol{d \;\textbf{sin} \;\theta = m +}

The equations for double slit interference imply that a series of bright and dark lines are formed. For vertical slits, the light spreads out horizontally on either side of the incident beam into a pattern called interference fringes, illustrated in Figure 6 . The intensity of the bright fringes falls off on either side, being brightest at the center. The closer the slits are, the more is the spreading of the bright fringes. We can see this by examining the equation

\boldsymbol{d \;\textbf{sin} \;\theta = m \theta , \;\textbf{for} \; m = 0, \;1, \; -1, \; 2, \; -2, \; \dots}.

Example 1: Finding a Wavelength from an Interference Pattern

\boldsymbol{10.95 ^{\circ}}

Substituting known values yields

$\begin{array}{r @{{}={}}l} \boldsymbol{\lambda} & \boldsymbol{\frac{(0.0100 \;\textbf{mm})(\textbf{sin} 10.95^{\circ})}{3}} \\[1em] & \boldsymbol{6.33 \times 10^{-4} \;\textbf{mm} = 633 \;\textbf{nm}}. \end{array}

Example 2: Calculating Highest Order Possible

Strategy and Concept

\boldsymbol{d \;\textbf{sin} \;\theta = m \lambda \; (\textbf{for} \; m = 0, \; 1, \; -1, \; 2, \; -2, \; \dots)}

The number of fringes depends on the wavelength and slit separation. The number of fringes will be very large for large slit separations. However, if the slit separation becomes much greater than the wavelength, the intensity of the interference pattern changes so that the screen has two bright lines cast by the slits, as expected when light behaves like a ray. We also note that the fringes get fainter further away from the center. Consequently, not all 15 fringes may be observable.

Section Summary

  • Young’s double slit experiment gave definitive proof of the wave character of light.
  • An interference pattern is obtained by the superposition of light from two slits.

\boldsymbol{d \;\textbf{sin} \;\theta = m \lambda \;(\textbf{for} \; m = 0, \; 1, \; -1, \;2, \; -2, \dots)}

Conceptual Questions

1: Young’s double slit experiment breaks a single light beam into two sources. Would the same pattern be obtained for two independent sources of light, such as the headlights of a distant car? Explain.

2: Suppose you use the same double slit to perform Young’s double slit experiment in air and then repeat the experiment in water. Do the angles to the same parts of the interference pattern get larger or smaller? Does the color of the light change? Explain.

3: Is it possible to create a situation in which there is only destructive interference? Explain.

4:   Figure 7 shows the central part of the interference pattern for a pure wavelength of red light projected onto a double slit. The pattern is actually a combination of single slit and double slit interference. Note that the bright spots are evenly spaced. Is this a double slit or single slit characteristic? Note that some of the bright spots are dim on either side of the center. Is this a single slit or double slit characteristic? Which is smaller, the slit width or the separation between slits? Explain your responses.

The figure shows a photo of a horizontal line of equally spaced red dots of light on a black background. The central dot is the brightest and the dots on either side of center are dimmer. The dot intensity decreases to almost zero after moving six dots to the left or right of center. If you continue to move away from the center, the dot brightness increases slightly, although it does not reach the brightness of the central dot. After moving another six dots, or twelve dots in all, to the left or right of center, there is another nearly invisible dot. If you move even farther from the center, the dot intensity again increases, but it does not reach the level of the previous local maximum. At eighteen dots from the center, there is another nearly invisible dot.

Problems & Exercises

2: Calculate the angle for the third-order maximum of 580-nm wavelength yellow light falling on double slits separated by 0.100 mm.

\boldsymbol{30.0 ^{\circ}}

7: At what angle is the fourth-order maximum for the situation in Problems & Exercises 1 ?

\boldsymbol{25.0 \;\mu \textbf{m}}

10: What is the smallest separation between two slits that will produce a second-order maximum for 720-nm red light?

11: (a) What is the smallest separation between two slits that will produce a second-order maximum for any visible light? (b) For all visible light?

\boldsymbol{10.0^{\circ}}

14: Using the result of the problem above, calculate the distance between fringes for 633-nm light falling on double slits separated by 0.0800 mm, located 3.00 m from a screen as in Figure 8 .

15: Using the result of the problem two problems prior, find the wavelength of light that produces fringes 7.50 mm apart on a screen 2.00 m from double slits separated by 0.120 mm (see Figure 8 ).

\boldsymbol{0.516 ^{\circ}}

9: 1200 nm (not visible)

11: (a) 760 nm

(b) 1520 nm

\boldsymbol{\textbf{sin} \;\theta - \;\textbf{tan} \;\theta \approx \theta}

For two adjacent fringes we have,

\boldsymbol{d \;\textbf{sin} \;\theta _{\textbf{m}} = m \lambda}

Subtracting these equations gives

$\begin{array}{r @{{}={}}l} \boldsymbol{d (\textbf{sin} \; \theta _{\textbf{m} + 1} - \textbf{sin} \; \theta _{\textbf{m}})} & \boldsymbol{[(m + 1) - m] \lambda} \\[1em] \boldsymbol{d(\theta _{{\textbf{m}} + 1} - \theta _{\textbf{m}})} & \boldsymbol{\lambda} \end{array}$

College Physics Copyright © August 22, 2016 by OpenStax is licensed under a Creative Commons Attribution 4.0 International License , except where otherwise noted.

Share This Book

IMAGES

  1. PPT

    young's single slit experiment class 12

  2. Visualization of the single-slit experiment. Received from

    young's single slit experiment class 12

  3. Single Slit Experiment Class 12 Notes

    young's single slit experiment class 12

  4. Single Slit Diffraction Experiment

    young's single slit experiment class 12

  5. Single Slit Diffraction: Young’s Single Slit Experiment and Fringe Width

    young's single slit experiment class 12

  6. Single Slit Experiment Class 12 Notes

    young's single slit experiment class 12

VIDEO

  1. Young's Double Slit Experiment

  2. Diffraction

  3. NEB Physics Class 12

  4. Wave Optics ~ Physics for Life Science Majors with Calculus

  5. Chapter 9, 9.4 Young's double slit experiment, First year physics

  6. class 12 : chapter 10 : d9 :Diffraction , single slit experiment , width of central maxima

COMMENTS

  1. What is diffraction of light

    Definition, Single Slit Diffraction, Formula, Video, and FAQs

  2. Diffraction of Light

    If the first dark fringe appears at an angle 30°, find the slit width. Solution: Using the diffraction formula for a single slit of width a, the nth dark fringe occurs for, a sin\ [\theta\] = nλ. At angle θ=30°, the first dark fringe is located. Using n=1 and. λ = 700 nm=700 X 10⁻⁹m, a sin 30°=1 X 700 X 10⁻⁹m.

  3. Physics Wave Optics part 20 (Young's Single slit Experiment) CBSE class 12

    Physics Wave Optics part 20 (Young's Single slit Experiment) CBSE class 12

  4. Single Slit Diffraction

    What is Single Slit Diffraction: Definition, Formula, Central ...

  5. Diffraction of Light at Single Slit, Chapter 10, Wave Optics, Class 12

    CBSE Exam, class 12. About Press Copyright Contact us Creators Advertise Developers Terms Privacy Policy & Safety How YouTube works Test new features NFL Sunday Ticket

  6. Single-Slit Diffraction: Derivation, Formula, and Pattern

    Single-Slit Diffraction: Derivation, Formula, and ...

  7. Single Slit Diffraction: Definition, Formula, Types and Examples

    Single Slit Diffraction: Definition, Formula, Types ...

  8. Physics Tutorial: Young's Experiment

    The previous section of Lesson 3 discussed Thomas Young's effort to derive an equation relating the wavelength of a light source to reliably measured distances associated with a two-point source light interference pattern. The equation, known as Young's equation is: λ = y • d / (m • L). In 1801, Young devised and performed an experiment to measure the wavelength of light.

  9. Single Slit Diffraction

    When light is incident on a slit with a width corresponding to the wavelength of light, a screen put in front of the slit produces an alternating dark and brilliant pattern. This is referred to as single slit diffraction. Young's Experiment with a Single Slit. In 1801, Thomas Young's double slit experiment demonstrated the wave structure of light.

  10. Single Slit Diffraction: Young's Single Slit Experiment and Fringe Width

    Key Terms: Diffraction, Single Slit Diffraction, Young's Single Slit Experiment, ... Class 11 & 12 Important Topics; NCERT Solutions for Class 12 Physics: Class 12 Physics Notes: Formulas in Physics: Topics for Comparison in Physics: NCERT Class 12 Biology Book: Topics with relation in physics:

  11. Single Slit Diffraction: Meaning, Formula, Types, Experiment

    Single Slit Diffraction: Meaning, Formula, Types, Experiment

  12. Diffraction from a single slit. Young's experiment with finite slits

    Diffraction pattern from a single slit. A laser illuminates a single slit and the resultant patten is projected on a distant screen. The sketch shows the view from above a single slit. Let's assume that the slit is constant width and very tall compared with that width, so that we can consider the system as two-dimensional.

  13. PDF Chapter 14 Interference and Diffraction

    14.2 Young's Double-Slit Experiment In 1801 Thomas Young carried out an experiment in which the wave nature of light was demonstrated. The schematic diagram of the double-slit experiment is shown in Figure 14.2.1. Figure 14.2.1 Young's double-slit experiment. A monochromatic light source is incident on the first screen which contains a slit .

  14. Compare Young's Double Slit Interference Pattern and Single Slit

    The angular width of the central maximum of the diffraction pattern in a single slit (of width a) experiment, with λ as the wavelength of light, is _____. In a double-slit experiment, green light (5303 Å) falls on a double slit having a separation of 19.44 µ-m and a width of 4.05 µm.

  15. Wave Interference

    Wave Interference

  16. Interference: Young's Double Slit Experiment

    Putting λ = 0.6 × 10 -6 m; D = 2m; d = 6 mm. y 3 =. Question of Class 12-Interference: Young's Double Slit Experiment : It was carried out in 1802 by the English scientist Thomas Young to prove the wave nature of light. Two slits S1 and S2 are made in an opaque screen, parallel and very close to each other. These two are illuminated by ...

  17. Double-slit experiment

    Double-slit experiment

  18. Lesson 58: Young's Double Slit Experiment

    Example 3: For a single slit experiment apparatus like the one described above, determine how far from the central fringe the first order violet (λ = 350nm) and red (λ = 700nm) colours will appear if the screen is 10 m away and the slit is 0.050 cm wide. We need to solve the formula for "x", the distance from the central fringe.

  19. 12.3 Young's Double Slit Experiment

    Figure 1. Young's double slit experiment. Here pure-wavelength light sent through a pair of vertical slits is diffracted into a pattern on the screen of numerous vertical lines spread out horizontally. Without diffraction and interference, the light would simply make two lines on the screen.

  20. Young's Double Slits Experiment Derivation

    The schematic diagram of the experimental setup is shown below-. Figure (1): Young double slit experimental set up along with the fringe pattern. A beam of monochromatic light is made incident on the first screen, which contains the slit S 0. The emerging light then incident on the second screen which consists of two slits namely, S 1, S 2.

  21. 27.3 Young's Double Slit Experiment

    The acceptance of the wave character of light came many years later when, in 1801, the English physicist and physician Thomas Young (1773-1829) did his now-classic double slit experiment (see Figure 1). Figure 1. Young's double slit experiment. Here pure-wavelength light sent through a pair of vertical slits is diffracted into a pattern on ...